5 6314347802025525707 PDF

You might also like

Download as pdf or txt
Download as pdf or txt
You are on page 1of 84

OFFICERS IAS ACADEMY

PREDICTION FOR UPSC CSE - PRELIMS 2020

ECONOMY ................................................................................................................................ 2
POLITY .....................................................................................................................................16
CURRENT AFFAIRS ................................................................................................................26
GEOGRAPHY...........................................................................................................................52
HISTORY ..................................................................................................................................65
ENVIRONMENT AND ECOLOGY ............................................................................................77
ART AND CULTURE ................................................................................................................82
OFFICERS IAS ACADEMY
(IAS Academy by IAS Officers)

ECONOMY
1. E-Kuber is related to
A. Digital initiative that aims to bring together farmers, PSUs, civil supplies and buyers on a
single platform
B. Portal to provide basic digital literacy to 10 lakh citizens in select blocks of each State/UT
C. Portal to deal with cross border insolvency
D. Core Banking Solution of the RBI

Answer: D

Explanation:
● E-Kuber is the Core Banking Solution of Reserve Bank of India. E-Kuber provides the
provision of a single current account for each bank across the country, with decentralised
access to this account from anywhere-anytime using portal-based services in a safe manner.
Application
● Auction of Government securities is done through the e-kuber system. Sovereign Gold
Bonds are available for subscription at the branches of scheduled commercial banks and
designated post offices through RBI’s e-kuber system.
● Goods and Service Tax (GST) settlements are also proposed to be done through e-
kuber. RBI has also launched a platform to enable trading in the priority sector lending
certificates (PSLC) through its Core Banking Solution (CBS) portal (e-Kuber).
What is Core Banking Solution?
● Core Banking Solutions (CBS) can be defined as a solution that enables banks to offer a
multitude of customer-centric services on a 24x7 basis from a single location, supporting
retail as well as corporate banking activities, as well as all possible delivery channels
existing and proposed.
● The centralisation thus makes a “one-stop” shop for financial services a reality. Using CBS,
customers can access their accounts from any branch, anywhere, irrespective of where they
have physically opened their accounts. Almost all branches of commercial banks, including
the Regional Rural Banks (RRBs), are brought into the core-banking fold.

2. With reference to the employment elasticity, consider the following statements:


1. It is a measure of the percentage change in employment associated with a 1 percentage
point change in economic growth.
2. Sectors with low employment elasticity are capable of absorbing a large workforce.

Which of the statements given above is/are correct?


A. 1 only
B. 2 only
C. Both 1 and 2
D. Neither 1 nor 2

Answer: A

Plot No: 935,6th Avenue, Anna Nagar, Chennai-40


Ph: 044-40483555,9677120226,9677174226
Web: www.officersiasacademy.com
OFFICERS IAS ACADEMY
(IAS Academy by IAS Officers)
Explanation:
● Statement 1 is correct: Employment elasticity is a measure of the percentage change
in employment associated with a 1 percentage point change in economic growth. The
employment elasticity indicates the ability of an economy to generate employment
opportunities for its population as per cent of its growth (development) process.
● Growth in sectors with high employment elasticity results in employment opportunities for
a large number of the population. Thus, statement 2 is incorrect.

3. Consider the following statements.


1. India was the leading country of origin of international migrants in 2019.
2. India remains as the top remittance recipient country in the world.

Which of the statements given above is/are correct?


A. 1 only
B. 2 only
C. Both 1 and 2
D. Neither 1 nor 2

Answer: C

Explanation:
● Statement 1 is correct: The Migration Report 2019 released by the United Nations has
placed India as the leading country of origin of international migrants in 2019 with a
diaspora strength of 17.5 million.
● Statement 2 is correct: As per the 2019 report of the World Bank, India remains as the top
remittance recipient country (with $79 billion), followed by China, Mexico, the Philippines
and Egypt.

4. The recommendations of Bimal Jalan committee constituted to resolve the tussle between
the RBI and the central government provided that:
1. RBI should maintain a Contingent Risk Buffer between 5.5-6.5% of the central bank’s
balance sheet.
2. Centre should invoke Section 7 of the RBI act frequently in public interest.
3. Casting vote in the Monetary Policy Committee should be transferred to the representative
of the government.

Which of the statements given above is/are correct?


A. 1 only
B. 1 and 2 only
C. 2 and 3 only
D. 1, 2 and 3 only

Answer: A

Explanation:

What constitutes RBI’s reserves?

Plot No: 935,6th Avenue, Anna Nagar, Chennai-40


Ph: 044-40483555,9677120226,9677174226
Web: www.officersiasacademy.com
OFFICERS IAS ACADEMY
(IAS Academy by IAS Officers)
● It comprises of 4 components out of which the Revaluation Fund and the
Contingency Fund form the substantial portion.
● Revaluation Fund- It is kept in terms of foreign currencies which is used to stabilize the
value of Indian Rupee during times of appreciation/depreciation.
● Contingency Fund is used to meet any risks in the Banking system.
● These 4 reserves constitute around 29% of the total assets of the RBI. The International
standard of reserves is 16% of the total assets of the central bank.

Bimal Jalan Committee


● The RBI’s reserves had been a long-standing issue between the Centre and the RBI. The
government had been putting pressure on the central bank to transfer more funds from the
RBI’s reserves.
● Bimal Jalan Committee on Economic Capital Framework was set up last year to look into the
central bank’s excess reserves and surplus transfer to the Centre.
● The committee has recently submitted its report. The committee recommended not to
transfer funds from the Revaluation account as it is critical to ensure stability of the Rupee.
● The committee recommended that the RBI should maintain a Contingent Fund
between 5.5-6.5% of the RBI’s total assets. Since the latest Contingent Fund amount was
about 6.8%, the excess amount (Rs 52,637 crore) was transferred to the government.
● In addition to this, the committee also recommended transferring the full net income of the
RBI of Rs 1,23,414 crore for the fiscal year 2018-19 to the Centre.
● Hence a total of Rs 1,76,051 crore has been paid out to the Centre. Hence, statement 1 is
correct.
● Remaining two statements are not related to the Bimal Jamal Committee.

About MPC
● The Monetary Policy Committee (MPC) is a committee of the RBI, headed by its Governor,
which is entrusted with the task of fixing the benchmark policy interest rate (repo rate) to
contain inflation within the specified target level.
● The MPC has six members
o RBI Governor (Chairperson), RBI Deputy Governor in charge of monetary policy,
one official nominated by the RBI Board and remaining 3 members would represent
the Government.
● The MPC makes decisions based on majority vote. In case of a tie, the RBI governor will have
a second or casting vote.

What does Section 7 of the RBI Act say?


● According to it, the Central government may issue directions to the RBI as it may “consider
necessary in public interest” after consultation with the RBI Governor. Section 7 deals with
‘management’ of RBI.

5. Consider the following statements about the Banking Ombudsman Scheme.


1. It is an expeditious forum to bank customers for resolution of complaints relating to certain
services rendered by banks.
2. All Scheduled Commercial Banks, Regional Rural Banks and Scheduled Primary Co-
operative Banks are covered under the Scheme.
3. One can approach the Ombudsman directly even without approaching the bank.

Plot No: 935,6th Avenue, Anna Nagar, Chennai-40


Ph: 044-40483555,9677120226,9677174226
Web: www.officersiasacademy.com
OFFICERS IAS ACADEMY
(IAS Academy by IAS Officers)
Which of the statements given above is/are correct?
A. 1 only
B. 1 and 3 only
C. 1 and 2 only
D. 1, 2 and 3

Answer: C

Explanation:
● Statement 1 is correct: Banking Ombudsman Scheme is an expeditious and inexpensive
forum to bank customers for resolution of complaints relating to certain services rendered
by banks.
● Statement 2 is correct: All Scheduled Commercial Banks, Regional Rural Banks and
Scheduled Primary Co-operative Banks are covered under the Scheme.
● Customers can approach Ombudsman for complaints relating to deficiency in banking
services like
○ Non-payment or inordinate delay in the payment or collection of cheques, drafts,
bills etc.
○ Non-acceptance, without sufficient cause, of small denomination notes or coins
tendered for any purpose, and for charging of commission in respect thereof;
○ Refusal to open deposit accounts without any valid reason for refusal;
○ Levying of charges without adequate prior notice to the customer;
● One's complaint will not be considered if he/she has not approached his bank for redressal
of his grievance first. Hence, statement 3 is incorrect.
● One can file a complaint before the Banking Ombudsman if the reply is not received from
the bank within a period of one month after the bank concerned has received one's
complaint, or the bank rejects the complaint, or if the complainant is not satisfied with the
reply given by the bank.
● To strengthen the grievance redressal mechanism for customers, RBI has tightened rules
concerning the banking ombudsman scheme and issued revised directions in the form of
‘Internal Ombudsman Scheme, 2018’.
● The IOS 2018 mandates Banks with more than 10 branches to have an Internal
Ombudsman to review complaints that were partially or wholly rejected by the respective
banks.
● It aimed to ensure that the complaints of the customers are redressed at the level of the
bank itself so as to minimize the need for the customers to approach other fora for
redressal.
● In 2018, the RBI also introduced an NBFC Ombudsman scheme to redress complaints with
regard to NBFCs.

6. With reference to the recent changes in the definition of Micro, Small & Medium
Enterprises (MSMEs) in India, consider the following statements.
1. MSMEs will be categorised based on the investment in machinery or equipment and annual
turnover.
2. The new definition has revised upwards the investment criteria of MSMEs.
3. The criteria for manufacturing units and service units will be the same for the classification
of MSMEs.

Plot No: 935,6th Avenue, Anna Nagar, Chennai-40


Ph: 044-40483555,9677120226,9677174226
Web: www.officersiasacademy.com
OFFICERS IAS ACADEMY
(IAS Academy by IAS Officers)
4. Units having investment between Rs 10 crore but up to Rs 50 crore and turnover
between Rs 50 crore and Rs 250 crore will now be known as Micro units.

Which among the above statements is/are correct?


A. 1, 2 and 3 only
B. 1, 2 and 4 only
C. 1, 3 and 4 only
D. 2, 3 and 4 only

Answer: A

Explanation:

● Statement 3 is correct: Recently, the government has changed the basic definition of
MSME and also end the difference between the manufacturing and services sector.
● Statements 1 and 2 are correct: Till now, MSMEs are categorised based only on the
investment in machinery or equipment. The new classification has raised the investment
limit and included annual turnover as an additional criterion.
○ Statement 4 is incorrect: Accordingly, units having investment less than Rs 1 crore
and turnover less than Rs 5 crore will be called Micro units.
○ Investment between Rs 1 and Rs 10 crore and turnover of Rs 5 crore to Rs 50 crore
will be categorised as Small Enterprises.
○ Units having investment between Rs 10 crore but up to Rs 50 crore and turnover
between Rs 50 crore and Rs 250 crore will now be known as Medium Enterprises.

Rationale for the move


● It has been a long-standing demand from industry to hike the investment limits, as with
inflation, units often cross the threshold that will bring them benefits. To prevent this, they
either run their operations at a reduced level or incorporate multiple units so that turnover
is distributed in a way that they remain within the threshold that will give them the
benefits.
● With the revised definitions of MSMEs, they will not have to worry about growing their size
and can still avail benefits.

7. Consider the following assets:


1. Certificates of Deposits
2. Demand deposits with Banks
3. Currency

The correct sequence of these assets in the decreasing order of liquidity is


A. 1-2-3
B. 2-3-1
C. 3-1-2
D. 3-2-1

Answer: D

Plot No: 935,6th Avenue, Anna Nagar, Chennai-40


Ph: 044-40483555,9677120226,9677174226
Web: www.officersiasacademy.com
OFFICERS IAS ACADEMY
(IAS Academy by IAS Officers)

Explanation:
● Liquid Assets are the assets of the business which can be converted into cash within a short
span of time.
● Currency is considered the most liquid asset among all the assets in the economy.
● A demand deposit is an account with a bank or other financial institution that allows the
depositor to withdraw his or her funds from the account without warning or with less than
seven days' notice. Thus, it also offers high liquidity for the depositors.
● Certificate of deposits (CDs) are money market instruments which can have a duration that
ranges from a few months to several years.

8. Free rider problem in economics refers


A. Burden on a shared resource created by non-paying users
B. Tax evasion practices prevailing in a gig economy
C. Transfer payments with low/zero value addition to the economy
D. Inflation in the country on account of government subsidies

Answer: A

Explanation:
● The free rider problem is the burden on a shared resource that is created by its use or
overuse by people who aren't paying their fair share for it or aren't paying anything at all.
● Their failure to contribute makes the resource economically infeasible to produce.
● The problem is commonly seen with public goods like access to clean air and water, flood
protection, free knowledge (public schools, libraries, and online educational content), roads,
bridges, sanitation regulations and so on.

9. With reference to the Remission of Duties or Taxes on Export Products (RoDTEP) scheme,
consider the following statements:
1. It will reimburse taxes and duties paid by exporters which are not getting exempted or
refunded under any other existing mechanism.
2. It will be a component of the Merchandise Exports from India Scheme (MEIS).

Which of the statements given above is/are correct?


A. 1 only
B. 2 only
C. Both 1 and 2
D. Neither 1 nor 2

Answer: A

Explanation:
● Statement 1 is correct: Recently, the Union Cabinet approved the Remission of Duties or
Taxes on Export Products (RoDTEP), a scheme for exporters to reimburse taxes and duties
paid by them such as value added tax, coal cess, mandi tax, electricity duties and fuel used
for transportation, which are not getting exempted or refunded under any other existing
mechanism.

Plot No: 935,6th Avenue, Anna Nagar, Chennai-40


Ph: 044-40483555,9677120226,9677174226
Web: www.officersiasacademy.com
OFFICERS IAS ACADEMY
(IAS Academy by IAS Officers)
● Statement 2 is incorrect: RoDTEP is set to replace the Merchandise Exports from
India Scheme (MEIS) as MEIS is not WTO-complaint.
● As and when the rates under the RoDTEP Scheme are announced for a tariff line/ item, the
Merchandise Exports from India Scheme (MEIS) benefits on such tariff line/item will be
discontinued.

10. With reference to liquidity trap, consider the following statements.


1. It is the situation where monetary policy becomes ineffective.
2. During liquidity trap, the demand for bonds increases.

Which of the statements given above is/are correct?


A. 1 only
B. 2 only
C. Both 1 and 2
D. Neither 1 nor 2

Answer: A

Explanation:
● Statement 1 is correct: A liquidity trap occurs when low / zero interest rates fail to
stimulate consumer spending and monetary policy becomes ineffective.
● Under normal conditions, when the supply of money in the economy increases and people
purchase bonds with this extra money, demand for bonds will go up. This will rise bond
prices and the rate of interest will decline (Bonds have an inverse relationship to interest
rates).
● However, if the market rate of interest is already low enough so that everybody expects it to
rise in the future, causing capital losses, nobody will wish to hold bonds. Hence, statement
2 is incorrect.
● Everyone in the economy will hold their wealth in money balance and if additional money is
injected within the economy it will be used up to satiate people’s craving for money
balances without increasing the demand for bonds and without further lowering the rate of
interest below. Such a situation is called a liquidity trap.

11. With reference to zero-coupon bonds, consider the following statements.


1. They are traded at its face value.
2. It does not pay interest to the bondholders.
3. Treasury bills are examples of zero-coupon bonds.

Which of the statements given above is/are correct?


A. 1 and 2 only
B. 1 and 3 only
C. 2 and 3 only
D. 1, 2 and 3

Answer: C

Explanation:

Plot No: 935,6th Avenue, Anna Nagar, Chennai-40


Ph: 044-40483555,9677120226,9677174226
Web: www.officersiasacademy.com
OFFICERS IAS ACADEMY
(IAS Academy by IAS Officers)
● Statement 1 is incorrect: A zero-coupon bond is a debt security that doesn't pay
interest (a coupon) but is traded at a price lower than its face value, rendering profit
at maturity when the bond is redeemed for its full face value.
● Statement 2 is correct: The difference between a regular bond and a zero-coupon bond, is
that the former pays bondholders interest, while the latter does not issue such interest
payments, otherwise known as coupons.
● Instead, zero-coupon bondholders merely receive the face value of the bond when it reaches
maturity, while coupon paying bondholders receive both the face value, while also receiving
coupons, over the life of the bond.
● Statement 3 is correct: Treasury bills are zero coupon securities and pay no interest.

12. With reference to the E-Way Bill system in GST, consider the following statements.
1. It is a mechanism to track the movement of goods by motorized and non-motorized
conveyance.
2. The objective is to check tax evasion.
3. An e-way bill is mandatory when the supply involves taxable goods and the value of a
consignment exceeds Rs. 50,000.

Which of the statements given above is/are correct?


A. 2 only
B. 1 and 2 only
C. 2 and 3 only
D. 1, 2 and 3

Answer: C

Explanation:
● Electronic Way Bill (E-Way Bill) is basically a compliance mechanism wherein by way of a
digital interface the person causing the movement of goods uploads the relevant
information prior to the commencement of movement of goods and generates e-way bill on
the GST portal.
● Statement 2 is correct: E-way bill is a mechanism to ensure that goods being transported
comply with the GST Law and is an effective tool to track the movement of goods and check
tax evasion.
● Statement 3 is correct: An e-way bill has to be prepared for every consignment where the
value of the consignment exceeds Rs.50,000/-.
● Exceptions to e-way bill requirement
○ goods being transported by a non-motorised conveyance; Hence, statement 1 is
incorrect.
○ goods being transported from the port, airport, air cargo complex and land customs
station to an inland container depot or a container freight station for clearance by
Customs;
● Contraceptives, judicial and non-judicial stamp paper, newspapers, khadi, raw silk, Indian
flag, human hair, kajal, earthen pots, cheques, municipal waste, puja samagri, LPG, kerosene,
etc. are also outside the ambit of the e-way bill.

Plot No: 935,6th Avenue, Anna Nagar, Chennai-40


Ph: 044-40483555,9677120226,9677174226
Web: www.officersiasacademy.com
OFFICERS IAS ACADEMY
(IAS Academy by IAS Officers)
13. With reference to Purchasing Power Parity (PPP) exchange rates, consider the
following statements.
1. They are calculated by comparing the prices of the same basket of goods and services in
different countries.
2. When two currencies are at purchasing power parity, goods cost the same in both countries,
taking into account the exchange rates.
3. In terms of PPP dollars, the U.S. is the largest economy in the world.

Which of the statements given above is/are correct?


A. 1 and 2 only
B. 2 and 3 only
C. 1 and 3 only
D. 1, 2 and 3

Answer: A

Explanation:
● Statement 1 is correct: Purchasing Power Parity (PPP) exchange rates are calculated by
comparing the prices of the same basket of goods and services in different countries.
● Statement 2 is correct: If the real exchange rate of two currencies is equal to one,
currencies are at purchasing power parity. This means that goods cost the same in both
countries, taking into account the exchange rates.
● For instance, if a pen costs $4 in the US and the nominal exchange rate is Rs 50 per US
dollar, then with a real exchange rate of 1, it should cost Rs 200 (50 × 4) in India. If the real
exchange rises above one, this means that goods abroad have become more expensive than
goods at home.
● Statement 3 is incorrect: In terms of PPP dollars, China is the largest economy in the
world.

14. Consider the following statements about Multilateral Convention to Implement Tax
Treaty Related Measures to Prevent Base Erosion and Profit Shifting.
1. It is an outcome of the United Nations Conference on Trade and Development (UNCTAD)
project to tackle Base Erosion and Profit Shifting.
2. The convention requires countries to modify tax treaties to curb revenue loss through
treaty abuse and base erosion and profit shifting strategies.
3. India has already ratified the convention.

Which among the above statements is/are correct?


A. 1 and 2 only
B. 2 only
C. 2 and 3 only
D. 1, 2 and 3

Answer: C

Explanation:

Plot No: 935,6th Avenue, Anna Nagar, Chennai-40


Ph: 044-40483555,9677120226,9677174226
Web: www.officersiasacademy.com
OFFICERS IAS ACADEMY
(IAS Academy by IAS Officers)
● Statement 1 is incorrect: The Multilateral Convention/MLI is an outcome of the
Organisation for Economic Co-operation and Development (OECD) / G20 Project to
tackle Base Erosion and Profit Shifting (the “BEPS Project”) i.e. tax planning strategies that
exploit gaps and mismatches in tax rules to artificially shift profits to low or no-tax locations
where there is little or no economic activity, resulting in little or no overall corporate tax
being paid.
● Statement 3 is correct: In 2019, India ratified the Convention, which was signed in 2017.
● Statement 2 is correct: The MLI will modify India’s tax treaties to curb revenue loss
through treaty abuse and base erosion and profit shifting strategies by ensuring that profits
are taxed where substantive economic activities generating the profits are carried out. The
MLI will be applied alongside existing tax treaties, modifying their application in order to
implement the BEPS measures.

15. The Essential Commodities (Amendment) Ordinance, 2020 seeks to deregulate which of
the following commodities?
1. Cereals
2. Edible oilseeds
3. Petroleum and its products
4. Fertilisers

Select the correct answer using the code given below


A. 1 and 2 only
B. 1 and 4 only
C. 1, 2 and 4 only
D. 1, 2, 3 and 4

Answer: A

Explanation:
● The Essential Commodities Act (ECA) was enacted by the Central Government in 1955 to
control and regulate trade and prices of commodities declared essential under the Act.
● The Act empowers the Central and state governments concurrently to control production,
supply and distribution of certain commodities in view of rising prices and to prevent Black
marketing.
● The measures that can be taken under the provisions of the Act include licensing,
distribution and imposing stock limits. The governments also have the power to fix price
limits, and selling the particular commodities above the limit will attract penalties.
● Most of the powers under the Act have been delegated by the Central Government to the
State Governments with the direction that they shall exercise these powers.
● Some of the major commodities that are covered under the act:
○ Petroleum and its products
○ Food stuff
○ Drugs- prices of essential drugs
○ Fertilisers

Plot No: 935,6th Avenue, Anna Nagar, Chennai-40


Ph: 044-40483555,9677120226,9677174226
Web: www.officersiasacademy.com
OFFICERS IAS ACADEMY
(IAS Academy by IAS Officers)
Essential Commodities (Amendment) Ordinance, 2020
● The Essential Commodities (Amendment) Ordinance, 2020 was promulgated on June
5, 2020. The Ordinance seeks to increase competition in the agriculture sector and enhance
farmers’ income. It aims to liberalise the regulatory system while protecting the interests of
consumers.
● The Ordinance provides that the central government may regulate the supply of certain
food items including cereals, pulses, potato, onions, edible oilseeds, and oils, only
under extraordinary circumstances. These include: (i) war, (ii) famine, (iii)
extraordinary price rise and (iv) natural calamity of grave nature.
● The Ordinance requires that imposition of any stock limit on certain specified items must be
based on price rise.
● A stock limit may be imposed only if there is: (i) 100% increase in retail price of
horticultural produce; and (ii) 50% increase in the retail price of non-perishable
agricultural food items. The increase will be calculated over the price prevailing
immediately preceding twelve months, or the average retail price of the last five years,
whichever is lower.
● The Ordinance provides that any stock limit will not apply to a processor or value chain
participant of agricultural produce if stock held by such person is less than the: (i)
overall ceiling of installed capacity of processing, or (ii) demand for export in case of an
exporter. A value chain participant means a person engaged in production, or in value
addition at any stage of processing, packaging, storage, transport, and distribution of
agricultural produce.
● The provisions of the Ordinance regarding the regulation of food items and the imposition
of stock limits will not apply to any government order relating to the Public
Distribution System or the Targeted Public Distribution System. Under these systems,
food grains are distributed by the government to the eligible persons at subsidised prices.

16. Tax buoyancy is defined as the


A. Opportunity cost of giving exemptions, deductions, rebates, etc. to the taxpayers
B. Ratio of growth in tax revenue to the growth in GDP
C. Ratio of growth in tax revenue to the growth in population
D. Ratio of gross tax collected to the total public debt of the state

Answer: B

Explanation:
● There is a strong connection between the government’s tax revenue earnings and economic
growth.
● Tax buoyancy explains this relationship. It is the ratio between the changes in government’s
tax revenue growth and the changes in GDP.
● It refers to the responsiveness of tax revenue growth to changes in GDP.
● When a tax is buoyant, its revenue increases with increase in GDP without increasing the
tax rate.

17. What will be the impact on the economy if RBI cuts down the cash reserve ratio?
1. It reduces the money multiplier.
2. It may increase investment in the economy.

Plot No: 935,6th Avenue, Anna Nagar, Chennai-40


Ph: 044-40483555,9677120226,9677174226
Web: www.officersiasacademy.com
OFFICERS IAS ACADEMY
(IAS Academy by IAS Officers)
3. Supply of currency in the economy may broaden.

Select the correct answer using the codes given below:


A. 1 and 2 only
B. 1 and 3 only
C. 2 and 3 only
D. 1, 2 and 3

Answer: C

Explanation:
● The RBI mandates that banks store a proportion of their deposits in the form of cash so that
the same can be given to the bank’s customers if the need arises. The percentage of cash
required to be kept in reserves, vis-a-vis a bank’s total deposits, is called the Cash Reserve
Ratio.
● Banks do not get any interest on the money that is with the RBI under the CRR
requirements.
● Reducing CRR provides more money in the hands of banks, which may now be lent out for
investment and increase the supply of currency in the economy. This has the potential
to spur investments in the country.
● Money multiplier is the amount of money the banking system generates out of each rupee
released by the central bank.
● It can be measured as: (1+c)/(c+r), where c is currency-deposit ratio and r is reserve
requirement ratio (Cash Reserve Ratio in India’s case).
● Thus, a country’s money multiplier depends on two factors—how much individuals (and
businesses) hold in cash and how much banks hold as reserves.
● The more individuals hold cash in hand, the less the banking system will be able to create
money and hence a lower value for the multiplier.
● Similarly, a higher reserve ratio reduces bank lending and therefore reduces the money
supply. A lower reserve ratio means a higher money multiplier.

18. Which of the following best describes the term “monetisation of deficit”?
A. Sovereign external borrowing from international institutions.
B. The government selling recapitalisation bonds in the financial system.
C. The government asking the Central bank to print new currency in return for government
bonds.
D. Triggering ‘escape clause’ provided in the FRBM Act.

Answer: C

Explanation:
● Monetisation of deficit is the monetary support the Reserve Bank of India (RBI) extends to
the Centre as part of the government's borrowing programme. It refers to the scenario
when the government asks the Central bank to print new currency in return for
government bonds.

Plot No: 935,6th Avenue, Anna Nagar, Chennai-40


Ph: 044-40483555,9677120226,9677174226
Web: www.officersiasacademy.com
OFFICERS IAS ACADEMY
(IAS Academy by IAS Officers)
● Also known as debt monetisation, the exercise leads to an increase in total money
supply in the system, and hence inflation, as RBI creates fresh money to purchase the
bonds. The same bonds are later used to bring down inflation as they are sold in the open
market. This helps RBI suck excess money out of the market and rein in rising prices.

19. Which of the following sectors are eligible for Priority Sector Lending (PSL)?
1. Education
2. Renewable Energy
3. Export Credit
4. Startup sector

Select the correct answer using the codes given below:


A. 1 only
B. 1 and 2 only
C. 1, 2 and 3 only
D. 1, 2, 3 and 4

Answer: D

Explanation:

● Priority Sector means those sectors which the Government of India and Reserve Bank of
India consider as important for the development of the basic needs of the country and are to
be given priority over other sectors. The banks are mandated to encourage the growth of
such sectors with adequate and timely credit.
● Priority Sector includes the following categories:
○ (i) Agriculture
○ (ii) Micro, Small and Medium Enterprises
○ (iii) Export Credit
○ (iv) Education
○ (v) Housing
○ (vi) Social Infrastructure
○ (vii) Renewable Energy
○ (viii) Others
● Targets Under the Priority Sector Lending
○ Domestic scheduled commercial banks (excluding Regional Rural Banks and Small
Finance Banks) and Foreign banks with 20 branches and above: 40 per cent of
Adjusted Net Bank Credit or Credit Equivalent Amount of Off-Balance Sheet
Exposure, whichever is higher.
○ Regional Rural Banks & Small finance banks: 75 per cent of Adjusted Net Bank
Credit or Credit Equivalent Amount of Off-Balance Sheet Exposure, whichever is
higher.

Plot No: 935,6th Avenue, Anna Nagar, Chennai-40


Ph: 044-40483555,9677120226,9677174226
Web: www.officersiasacademy.com
OFFICERS IAS ACADEMY
(IAS Academy by IAS Officers)
○ Foreign banks with less than 20 branches: 40 per cent of Adjusted Net Bank
Credit or Credit Equivalent Amount of Off-Balance Sheet Exposure, whichever
is higher, to be achieved in a phased manner by 2020.
○ Sub-targets are specified for certain sectors like 18% to agriculture with 8% to
small and marginal farmers, 7.5% to micro units etc.
● Recently, the RBI brought startups under the purview of priority sector lending, a move
that will make it easier for startups to raise funds from banks.

20. Consider the following sectors.


1. Petroleum products
2. Marine products
3. Pearls, precious and semi-precious stones
4. Iron and steel

Arrange them in decreasing order in terms of their share in India’s export basket in 2019-
20.
A. 1-2-3-4
B. 1-3-4-2
C. 2-1-3-4
D. 3-1-2-4
Answer: B
Explanation:

Plot No: 935,6th Avenue, Anna Nagar, Chennai-40


Ph: 044-40483555,9677120226,9677174226
Web: www.officersiasacademy.com
OFFICERS IAS ACADEMY
(IAS Academy by IAS Officers)

POLITY
1. Consider the following statements relating to Chief Minister of a state

1. The Chief Minister can either be from Legislative assembly or legislative council
2. A nominated member cannot become Chief Minister of a state
Which of the above statements is/are correct?
a) 1 ONLY
b) 2 only
c) Both 1 nor 2
d) Neither 1 nor 2

Answer : a

Explanation

A Minister including Chief Minister of a state can be an MLA or MLC (examples of MLC being CM are
Mr Yogi Adityanath, Mr Nitish Kumar)

Any person can become a CM but he has to be become a member of Legislative assembly or council
within 6 months. Even a nominated member can become CM or any other minister.
This issue is relevant under present circumstances when there was controversy surrounding the
nomination of Mr Uddhav Thackarey to Legislative council.

2. Consider the following statements with respect to the Tenth Schedule of the constitution
dealing with disqualification of members
1. The Speaker or the Chairman is the appropriate authority for deciding on disqualification of
members from the house.
2. A disqualified member cannot be appointed as a Minister until his re-election
3. Unless there are exceptional circumstances, any petition for disqualification should be
decided by the speaker within a period of 3 months.
Which of the above statements is/are correct?
a) 1 only
b) 1 and 2 only
c) 1 and 3 only
d) 1, 2 and 3
Answer: D

Explanation

Plot No: 935,6th Avenue, Anna Nagar, Chennai-40


Ph: 044-40483555,9677120226,9677174226
Web: www.officersiasacademy.com
OFFICERS IAS ACADEMY
(IAS Academy by IAS Officers)
As per Article 75(1B) and 164(1B), a disqualified member cannot become a minister till
getting re-elected. So, statement 2 is correct
As per recent SC judgement, unless there are exceptional circumstances, speaker should decide
within 3 months. Hence it is a SC mandated law and hence statement 3 is also correct.

3. Consider the following statements with respect to National Human Rights commission
1. The Chairperson of National Human Rights commission shall be a person who has been the
Chief Justice of India
2. The tenure of members of National Human Rights commission is 3 years
Which of the above statements is/are correct?
a) 1 ONLY
b) 2 only
c) Both 1 nor 2
d) Neither 1 nor 2

Answer : B

Explanation

The NHRC act has been amended recently with the following modifications:

 that a person who has been a Judge of the Supreme Court is also made eligible to be
appointed as Chairperson of the Commission in addition to the person who has been the
Chief Justice of India;
 to increase the Members of the Commission from two to three of which, one shall be a
woman;
 to include Chairperson of the National Commission for Backward Classes, Chairperson of
the National Commission for Protection of Child Rights and the Chief Commissioner for
Persons with Disabilities as deemed Members of the Commission;
 to reduce the term of the Chairperson and Members of the Commission and the State
Commissions from five to three years and shall be eligible for re-appointment;
 to provide that a person who has been a Judge of a High Court is also made eligible to be
appointed as Chairperson of the State Commission in addition to the person who has been
the Chief Justice of the High Court; and,
 to confer upon State Commissions, the functions relating to human rights being discharged
by the Union territories, other than the Union territory of Delhi, which will be dealt with by
the Commission.

4. Which of the following criteria has been added newly by the 15th Finance commission for
devolution of the share of net tax proceeds amongst the states.
a) Income distance
b) Population
c) Area
d) Demographic performance

Plot No: 935,6th Avenue, Anna Nagar, Chennai-40


Ph: 044-40483555,9677120226,9677174226
Web: www.officersiasacademy.com
OFFICERS IAS ACADEMY
(IAS Academy by IAS Officers)

Answer : D

Explanation

Key recommendations of 15th Finance commission

 Devolution of taxes to states: The share of states in the centre’s taxes is recommended to be
decreased from 42% during the 2015-20 period to 41% for 2020-21. The 1% decrease is to
provide for the newly formed union territories of Jammu and Kashmir, and Ladakh from the
resources of the central government.
Criteria for Devolution – (figures in brackets were the weightage as per 14th FC)
Income distance – 45% (50%)
Population (2011) – 15% (10%)
Area – 15% (15%)
Forest and Ecology – 10% (7.5%)
Demographic performance – 12.5% (new criteria to replace 1971 census which had 10%
weightage)
Tax effort – 2.5% (new criteria)

5. Which of the following states do not have any special provisions under the Indian
constitution?
a) Nagaland
b) Manipur
c) Mizoram
d) Tripura
Answer : D

Explanation

Pl refer articles 371A to 371J under the constitution. In the wake of scrapping of Article 370, these
special provisions may be tested in the UPSC prelims.

6. The National Eligibility Entrance Test (NEET) for medical admissions is applicable to
which of the following Medical educational institutions
1. Government Medical colleges
2. Private medical colleges
3. Minority run medical colleges

Plot No: 935,6th Avenue, Anna Nagar, Chennai-40


Ph: 044-40483555,9677120226,9677174226
Web: www.officersiasacademy.com
OFFICERS IAS ACADEMY
(IAS Academy by IAS Officers)
Select the correct answer from codes below

a) 1 only
b) 1 and 2 only
c) 1 and 3 only
d) 1, 2 and 3

Answer : D

Explanation

Recently the SC gave a judgement that NEET is applicable to minority educational institutions as
well and it does not violate their rights under Article 30. The right to religion and minority
educational rights are not absolute imposing reasonable restrictions to ensure educational
standards was in national and public interest.

7. Which of the following changes would require an amendment to the constitution under
Article 368?
a) Formation of a new state or a union territory and consequent amendment to Schedule 1
b) Amendment to Schedule 4 consequent upon formation of a new state or union territory
c) Amendment by way of addition, variation or repeal of any of the provisions of Schedule 5
d) Placing a law passed by Parliament or State legislature in the ninth schedule
Answer : D

Explanation

The first two statements require only an act of parliament – Recent instances being J & K and in
2014, Telengana
The amendment to Schedule 5 and Schedule 6 can be through a law of parliament and is not
considered an amendment under Art 368
Placing of any law in Ninth schedule requires a constitutional amendment. Such a law is still
subject to judicial review. It was in news recently as law for reservation for SC/STs in promotions
was demanded to be included in ninth schedule through constitutional amendment. Hence option
D is the correct answer.

8. In the context of liberty, what does the term ‘Harm principle’ result in

a) The use of power over any member of a community to prevent harm to others.
b) The actions of any member of a community which inflict harm on themselves
c) The actions of any member of a community which inflict harm on the community
d) The use of power over any member of a community to cause harm to such member.

Answer : a

Plot No: 935,6th Avenue, Anna Nagar, Chennai-40


Ph: 044-40483555,9677120226,9677174226
Web: www.officersiasacademy.com
OFFICERS IAS ACADEMY
(IAS Academy by IAS Officers)
Explanation

Harm principle was introduced by J S Mill in his book ‘On Liberty’. It means that harm principle,
should be the criteria for imposing restrictions on freedom/liberty to others. If any act of a
member of a society can cause serious harm to others, then their actions can be restricted. This is
reflected through ‘reasonable restrictions’ in the Indian constitution.
Refer Chapter ‘Freedom’ in NCERT, Political theory book.
This is important considering the protests against CAA and whether the Shaheen Bagh protests can
be continued considering that it affected the movement of other members of the society.

9. Which of the following best defines citizenship?


a) A give and take where state confers rights on its citizens and citizens have duties towards the
state
b) Citizenship is the right to have rights
c) Citizenship is the relationship that gives the right to vote in elections
d) Citizenship is the guarantee of social and economic rights to citizens
Answer : b

Explanation

This is a famous quote by political thinker Hannah Arendt. Citizenship is the right to enjoy various
fundamental and other rights. Duties are ideals that a citizen should follow but cannot be the basis
for citizenship. Hence option b is the right answer.

10. With reference to the Environmental Impact Assessment Notification recently seen in
news, which of the following statements is/are correct?

1. It is framed by the Central Government under the Environment (Protection) Act, 1986.
2. Forests as an entry is part of the Union list in the Seventh Schedule which enables the Central
Government to notify the same.
Select the correct answer using the code given below.
(a) 1 only
(b) 2 only
(c) Both 1 and 2
(d) Neither 1 nor 2

Answer: (a)

Explanation:

Plot No: 935,6th Avenue, Anna Nagar, Chennai-40


Ph: 044-40483555,9677120226,9677174226
Web: www.officersiasacademy.com
OFFICERS IAS ACADEMY
(IAS Academy by IAS Officers)
Under the Environment (Protection) Act, 1986, India notified its first EIA norms in 1994,
setting in place a legal framework for regulating activities that access, utilise, and affect
(pollute) natural resources. Every development project has been required to go through the EIA
process for obtaining prior environmental clearance ever since.
The 1994 EIA notification was replaced with a modified draft in 2006. Earlier this year, the
government redrafted it again to incorporate the amendments and relevant court orders issued
since 2006, and to make the EIA “process more transparent and expedient.” Hence statement 1 is
correct

Forests was originally part of State list and was moved to Concurrent list by 42nd amendment Act of
1976. Hence Statement 2 is wrong.

11. Which of the following languages were added to the Eighth schedule through
amendments to the constitution (after 1950)
1. Manipuri
2. Nepali
3. Sindhi
4. Maithili

Choose the correct answer using codes given below


a) 1 and 2 only
b) 1, 2 and 3 only
c) 2 and 4 only
d) 1, 2, 3 and 4

Ans: (d)

Explanation

14 languages were initially included in the Constitution. Subsequently, Sindhi was added in 1967 by
21st constitutional amendment act; Konkani, Manipuri and Nepali were added in 1992 by 71st
Constitutional Amendment Act; and Bodo, Dogri, Maithili and Santali were added in 2004 by 92nd
Constitutional Amendment Act.
Considering the two-language vs three-language formula that has been debated in the New
Education Policy, this may be a potential question.

12. Who is competent to prescribe conditions as for acquiring Indian citizenship?


a) Union Parliament
b) State legislatures
c) President of India
d) Union Parliament in consultation with state legislatures

Plot No: 935,6th Avenue, Anna Nagar, Chennai-40


Ph: 044-40483555,9677120226,9677174226
Web: www.officersiasacademy.com
OFFICERS IAS ACADEMY
(IAS Academy by IAS Officers)
Ans : (a)

Explanation

Citizenship is a Union list subject and as per Article 11, Parliament has the power to regulate
citizenship. Hence option (a) is the correct answer. Recently CAA was passed by Parliament.

13. Which of the following falls under the ‘Original jurisdiction’ of the Supreme court?
1. Dispute between Union Government and a State Government
2. Dispute between two State governments
3. Dispute between Union Government and a Union territory having an elected government

Choose the answer from the codes given below


a) 1 only
b) 2 only
c) 1 and 2 only
d) 1, 2 and 3
Answer : (c)

Explanation

Refer Art 131 – The first two disputes fall under category of Original jurisdiction. Hence Statement
1 and 2 are correct.
Dispute between Union Government and UT of Delhi and Puducherry go to respective High courts
first as they are not in the realm of original jurisdiction under Art 131. Hence statement 3 is
incorrect.
Recently Kerala govt has moved SC against CAA but it is yet to be considered by SC. There were also
disputes between Union Govt and Puducherry govt which was adjudicated by Madras HC recently.

14. The conditions of service and tenure of Comptroller and Auditor General is prescribed by
a) The President
b) The Constitution of India
c) The Central Government
d) The Parliament
Answer : (d)

Explanation

Pl refer Art 148 and also CAG Act, 1971. Same is the case with Election commissioners also.
Whereas retirement age of SC and HC judges is provided in the Constitution itself.
Recently CAG was appointed and also a new EC was appointed.

Plot No: 935,6th Avenue, Anna Nagar, Chennai-40


Ph: 044-40483555,9677120226,9677174226
Web: www.officersiasacademy.com
OFFICERS IAS ACADEMY
(IAS Academy by IAS Officers)

15. How many members of the Lok Sabha need to support a No-Confidence motion for it
to be admitted by the Speaker
a) One-fourth of total membership of the house
b) One-tenth of total membership of the house
c) One-tenth of members present in the house on the day of admission of motion
d) Fifty members of Lok Sabha
Answer : (d)

Explanation

 As per rules of Lok Sabha – If 50 members support admission of no-confidence motion, it


should be admitted by the Speaker and it should be discussed and put to vote within 10
days.
 No reason needs to be given for introduction of No-confidence motion and it is against the
entire council of ministers
 If a no-confidence motion is defeated then no new no-confidence motion can be introduced
for 6 months.
 A No-confidence motion was moved in Lok Sabha after 15 years in July 2018.

16. A judge of a High court wanting to resign addresses his letter of resignation to
a) The Chief justice of India
b) The Chief justice of the respective High court
c) The Governor of the State
d) The President of India

Answer : (d)

Explanation

SC and HC judges are appointed by President, removed by Parliament and they submit
resignation to the President of India.

In September 2019, CJ of Madras HC Ms. Tahilramani had resigned after a controversy.

17. Which of the following feature distinguishes Rajya Sabha from the legislative council of a
state?
a) Tenure of membership of each member
b) System of election
c) Nomination of members to the house
d) Participation in President election

Answer : (d)

Plot No: 935,6th Avenue, Anna Nagar, Chennai-40


Ph: 044-40483555,9677120226,9677174226
Web: www.officersiasacademy.com
OFFICERS IAS ACADEMY
(IAS Academy by IAS Officers)

Explanation

Tenure of membership is same at 6 years; system of election is same through proportional


representation; both the houses have nominated members as per provisions of constitution.
However, while elected Rajya Sabha members participate in President elections, the members of
legislative council don’t vote in president elections. Hence option (d) is correct answer.
Recently constitution 104th amendment removed nominated Anglo- Indian members to Lok Sabha
and Legislative assemblies.

18. Which of the following directive principles is yet to be implemented by the State?
(a) Uniform civil code
(b) Separation of Judiciary from Executive
(c) Right to work
(d) Equal pay for equal work for both men and women
Answer (a)
Explanation

UCC is yet to be implemented and lot of discussion about the same. Hence option (a) is correct. The
other three have been implemented
Separation of judiciary from executive – largely done through CrPC 1973
Right to work – MGNERA, 2005
Equal pay for equal work – Equal remuneration Act 1976 and now it has been replaced by Code on
Wages 2019.

19. Consider the Following Statements –


1) The Supreme Court has the power to punish for the contempt of Court.
2) The Original Jurisdiction of the Supreme Court includes its power to preside over disputes
between Centre and State and Inter-State issues.
Which of the Statements given above is/are correct?
a) 1 only
b) 2 only
c) Both 1 and 2
d) Neither 1 and 2
Answer- (c)

Explanation

The Supreme Court is also a court of record and shall have all the powers of such a court including
the power to punish for contempt of itself. Statement 1 is correct.

Plot No: 935,6th Avenue, Anna Nagar, Chennai-40


Ph: 044-40483555,9677120226,9677174226
Web: www.officersiasacademy.com
OFFICERS IAS ACADEMY
(IAS Academy by IAS Officers)
The Supreme Court shall have original jurisdiction in any dispute, which is—
(a) between the Government of India and one or more States; or
(b) between the Government of India and any State or States on one side and one or more other
States on the other; or
(c) between two or more States.
Hence, Statement 2 is correct, as it includes Centre-State and Inter-State disputes.
Source – Indian Constitution, Part V- The Union, Article -129 and 131.

20. Consider the following statements about the National Commission for Backward Classes

1. It is a constitutional body set up under the 102nd amendment act to the Constitution of India.
2. Prior to the constitutional amendment, the National Commission for Backward classes was a
Statutory body under National Commission for Backward classes Act, 1993
3. It will be responsible for monitoring safeguards for backward classes and enquiring into
complaints of their rights violated.
4. It will also be responsible for adding or removing a caste name from the Central list of
backward classes.

Which of the statements given above are correct?

a)1 and 2 only


b)1 and 3 only
c) 1, 2 and 3 only
d)1, 2, 3 and 4

Answer : (c)

Explanation

The first three statements are all correct as per constitutional amendment and under Art 338B.
However, statement 4 is wrong as it is Parliament which will be responsible for adding or
removing a caste name from central list of backward classes.

Plot No: 935,6th Avenue, Anna Nagar, Chennai-40


Ph: 044-40483555,9677120226,9677174226
Web: www.officersiasacademy.com
OFFICERS IAS ACADEMY
(IAS Academy by IAS Officers)

CURRENT AFFAIRS
1. Which one of the following best describes the “principle of non refoulement” sometimes
seen in the news?
A. It is a legal doctrine which states that once a right is recognised and given to the public, it
cannot be taken back by the state at a later date
B. It forbids a country receiving asylum seekers from returning them to a country in which
they would be in likely danger of persecution
C. Criterion used by countries to prevent undue exploitation of benefits of a preferential
system
D. It prohibits discrimination between imported and domestically produced goods with
respect to internal taxation or other government regulation

Answer: B

Explanation:
● Under international human rights law, the principle of non-refoulement guarantees that no
one should be returned to a country where they would face torture, cruel, inhuman or
degrading treatment or punishment and other irreparable harm. This principle applies to all
migrants at all times, irrespective of migration status.
● Option A is related to the “Doctrine of Progressive Realisation of Rights”
● Option C is related to the “Rule of Origin”
● Option D is related to the “National treatment” clause of the WTO

2. ‘Operation Twist’ is related to


A. National level strategy for implementation of programmes to tackle the air pollution
B. Promotion of physical culture and sports among youths and children
C. Multilateral naval exercise hosted by the Indian Navy under the aegis of the Andaman and
Nicobar Command
D. Simultaneous purchase and sale of government securities by the RBI

Answer: D

Explanation
● The Reserve Bank of India conducted its version of ‘Operation Twist’ in December 2019.
● It involved simultaneous purchase and sale of government securities worth Rs 10,000 crore
and Rs 6,825 crore respectively under Open Market Operations.
● Option C is related to Milan Exercise.

3. Consider the following statements about the Singapore Convention on Mediation


1. It is a binding international instrument for the facilitation of international trade
2. India is a signatory to the convention

Plot No: 935,6th Avenue, Anna Nagar, Chennai-40


Ph: 044-40483555,9677120226,9677174226
Web: www.officersiasacademy.com
OFFICERS IAS ACADEMY
(IAS Academy by IAS Officers)

Which of the statements given above is/are correct?


A. 1 only
B. 2 only
C. Both 1 and 2
D. Neither 1 nor 2

Answer: C

Explanation
● The United Nations General Assembly adopted the United Nations Convention on
International Settlement Agreements Resulting from Mediation (Also known as the
"Singapore Convention on Mediation”) in 2018.
● Statement 1 is correct: The Convention is an instrument for the facilitation of international
trade and the promotion of mediation as an alternative and effective method of resolving
trade disputes. Being a binding international instrument, it is expected to bring certainty
and stability to the international framework on mediation, thereby contributing to the
Sustainable Development Goals (SDG), mainly the SDG 16.
● Statement 2 is correct: In 2019, the Union Cabinet has approved the signing of the United
Nations Convention on International Settlement Agreements.

4. Which among the following schemes/initiatives follow a 6X6X6 strategy to address


malnutrition?
A. Integrated Child Development Services
B. Anemia Mukt Bharat
C. POSHAN Abhiyaan
D. Public Distribution System

Answer: B

Explanation:
● The Anemia Mukt Bharat initiative aims to strengthen the existing mechanisms and foster
newer strategies for tackling anemia.
● It follows a 6X6X6 strategy (six target beneficiary groups, through six interventions and six
institutional mechanisms) to reduce prevalence of anemia by 3 percentage points per year.

Plot No: 935,6th Avenue, Anna Nagar, Chennai-40


Ph: 044-40483555,9677120226,9677174226
Web: www.officersiasacademy.com
OFFICERS IAS ACADEMY
(IAS Academy by IAS Officers)

● The six population groups under AMB strategy are:


○ Children (6-59 months)
○ Children (5-9 years)
○ Adolescents girls and boys (10-19 years)
○ Pregnant women
○ Lactating women
○ Women of Reproductive Age (WRA) group (15-49 years)
● In the “State of the World’s Children 2019” report, UNICEF has commended the Anaemia
Mukt Bharat programme as one of the best programmes implemented by governments
across the world to address malnutrition.

5. Consider the following schemes


1. Pradhan Mantri Shram Yogi Maan-dhan
2. Pradhan Mantri Laghu Vyapari Maan-dhan Yojana
3. Pradhan Mantri JI-VAN Yojana
4. Pradhan Mantri Vaya Vandana Yojana

Which of the above schemes aim to increase insurance cover in the country?
A. 1 and 2 only
B. 3 and 4 only
C. 1, 2 and 4 only
D. 1, 2, 3 and 4

Answer: C

Explanation

About Pradhan Mantri Shram Yogi Maan-dhan

Plot No: 935,6th Avenue, Anna Nagar, Chennai-40


Ph: 044-40483555,9677120226,9677174226
Web: www.officersiasacademy.com
OFFICERS IAS ACADEMY
(IAS Academy by IAS Officers)
● It is a Central Sector pension scheme to ensure old age protection for Unorganised
Workers.
● It is administered by the Ministry of Labour and Employment and implemented through
Life Insurance Corporation of India and Common Service Centres (CSCs).
● LIC will be the Pension Fund Manager and responsible for Pension pay out.

Eligibility:
● The Unorganised workers whose monthly income is Rs 15,000/ per month or less and
belong to the entry age group of 18-40 years are eligible for the scheme.
● They should not be covered under New Pension Scheme (NPS), Employees’ State Insurance
Corporation (ESIC) scheme or Employees’ Provident Fund Organisation (EPFO).
● He/she should not be an income tax payer.

Salient Feature of PM-SYM:


● Matching contribution by the Central Government: PM-SYM is a voluntary and
contributory pension scheme on a 50:50 basis where prescribed age-specific contribution
shall be made by the beneficiary and the matching contribution by the Central Government.
● Minimum Assured Pension: Each subscriber under the PM-SYM, shall receive a minimum
assured pension of Rs 3000/- per month after attaining the age of 60 years.
● Family Pension: During the receipt of pension, if the subscriber dies, the spouse of the
beneficiary shall be entitled to receive 50% of the pension received by the beneficiary as
family pension. Family pension is applicable only to the spouse.

Pradhan Mantri Laghu Vyapari Maan-dhan Yojana


● The National Pension Scheme for Traders and Self-Employed Persons Yojana (also known
as Pradhan Mantri Laghu Vyapari Maan-dhan Yojana) is a pension scheme for
shopkeeper’s/ retail traders and self-employed persons.
● It is a voluntary and contribution based central sector scheme which provides a monthly
minimum assured pension of Rs 3000/- for eligible beneficiaries.
Eligibility
● The scheme is open traders and self employed persons within the age group of 18-40
years.
● Small traders whose annual turnover does not exceed Rs 1.5 crore.
● The following are not eligible to join the Scheme
○ If covered under National Pension Scheme/ Employees’ State Insurance Corporation
Scheme/ Employees’ Provident Fund Scheme/ Pradhan Mantri Shram Yogi
Maandhan
○ Is an income-tax assessee.

Pradhan Mantri Vaya Vandana Yojana (PMVVY)

● It is a pension scheme available for senior citizens. The scheme offers a guaranteed payout
of pension at a specified rate for 10 years.
● It also offers a death benefit in the form of return of purchase price to the nominee.

Eligibility

Plot No: 935,6th Avenue, Anna Nagar, Chennai-40


Ph: 044-40483555,9677120226,9677174226
Web: www.officersiasacademy.com
OFFICERS IAS ACADEMY
(IAS Academy by IAS Officers)
● Minimum entry age for this scheme is 60 years. There is no maximum age limit.
● The maximum investment allowed per person is Rs 15 lakh.

Pradhan Mantri JI-VAN Yojana


● The "Pradhan Mantri JI-VAN (Jaiv Indhan- Vatavaran Anukool fasal awashesh Nivaran)
Yojana" aims at providing financial support to Integrated Bioethanol Projects using
lignocellulosic biomass and other renewable feedstock.
● Under this Yojana, 12 Commercial Scale and 10 demonstration scale Second Generation
(2G) ethanol Projects will be provided with Viability Gap Funding (VGF) support in two
phases.

6. Consider the following statements about the poliovirus


1. Two of the three wild poliovirus strains are now eradicated globally.
2. Inactivated Poliovirus Vaccine does not cause Vaccine Associated Paralytic Poliomyelitis
(VAPP) or circulating vaccine-derived poliovirus (cVDPV).

Which of the statements given above is/are correct?


A. 1 only
B. 2 only
C. Both 1 and 2
D. Neither 1 nor 2

Answer: C

Explanation:
What is Polio?
▪ Poliomyelitis (polio) is a highly infectious viral disease caused by the Polio virus which
mainly affects young children.
▪ The virus is transmitted by person-to-person spread mainly through the faecal-oral route
or, less frequently, by a common vehicle (e.g. contaminated water or food) and multiplies in the
intestine, from where it can invade the nervous system.
▪ There is no cure for polio, it can only be prevented by immunization.
About Polio vaccine:
▪ There are three wild types of poliovirus (WPV) – type 1, type 2, and type 3.
▪ There are two vaccines used to protect against polio disease, oral polio vaccine (OPV) and
inactivated poliovirus vaccine (IPV).
▪ After wild poliovirus type 2 was declared eradicated in 2015, the world switched from
trivalent to bivalent OPV. Trivalent OPV contains all three types of poliovirus, while bivalent OPV
only contains poliovirus type 1 and 3.
Vaccine derived Polio:
▪ Oral polio vaccine contains attenuated (weakened) vaccine-virus, activating an immune
response in the body.
▪ When a child is immunized with OPV, the weakened vaccine-virus replicates in the intestine
for a limited period, thereby developing immunity by building up antibodies.
▪ During this time, the vaccine-virus is also excreted. In areas of inadequate sanitation, this
excreted vaccine-virus can spread in the immediate community, before eventually dying out.

Plot No: 935,6th Avenue, Anna Nagar, Chennai-40


Ph: 044-40483555,9677120226,9677174226
Web: www.officersiasacademy.com
OFFICERS IAS ACADEMY
(IAS Academy by IAS Officers)
▪ OPV is very effective against the wild poliovirus, but in very rare cases the vaccine can
lead to paralysis. One type of polio caused by OPV is called Vaccine Associated Paralytic
Poliomyelitis (VAPP).
▪ Another form of vaccine associated polio is the Circulating Vaccine Derived Poliovirus
(cVDPV). These are mutated versions of OPV which can cause paralysis and spread from person-
to-person.
▪ Statement 2 is correct: Inactivated Poliovirus Vaccine does not cause Vaccine Associated
Paralytic Poliomyelitis (VAPP) or circulating vaccine-derived poliovirus (cVDPV) as it doesn’t
contain live viruses.

Why in News?
▪ Statement 1 is correct: The Global Commission for the Certification of Poliomyelitis
Eradication officially declared that wild poliovirus type 3 has been eradicated globally.
▪ With two of the three wild polioviruses eliminated, only type 1 wild poliovirus is still in
circulation and is restricted to just two countries — Afghanistan and Pakistan.
▪ It opens up the possibility of switching from the currently used bivalent oral polio vaccine
containing type 1 and type 3 to a monovalent vaccine containing only type 1.

7. Consider the statements about Atal Bhujal Yojana


1. It is a pan India initiative of the Department of Water Resources, River Development &
Ganga Rejuvenation.
2. The chief objective of the Mission is to provide Functional Household Tap Connection
(FHTC) to every rural household by 2024.

Which of the statements given above is/are correct?


A. 1 only
B. 2 only
C. Both 1 and 2
D. Neither 1 nor 2

Answer: D

Explanation
● Both statements are incorrect:
● Atal Bhujal Yojana (or, Atal Jal) has been designed with the principal objective of
strengthening the institutional framework for participatory groundwater management
and bringing about behavioral changes at the community level for sustainable
groundwater resource management in seven States, viz. Gujarat, Haryana, Karnataka,
Madhya Pradesh, Maharashtra, Rajasthan and Uttar Pradesh. (not a pan India initiative).
● The scheme envisages undertaking various interventions, including awareness
programmes, capacity building, convergence of ongoing/new schemes and improved
agricultural practices etc.
● It is implemented by the Department of Water Resources, River Development & Ganga
Rejuvenation under the Ministry of Jal Shakti.
● Jal Jeevan Mission has the chief objective of providing Functional Household Tap
Connection (FHTC) to every rural household by 2024.

Plot No: 935,6th Avenue, Anna Nagar, Chennai-40


Ph: 044-40483555,9677120226,9677174226
Web: www.officersiasacademy.com
OFFICERS IAS ACADEMY
(IAS Academy by IAS Officers)

8. Consider the following statements.


1. The National Disaster Management Authority is a statutory body.
2. The Union Home Minister acts as the chairman of the National Disaster Management
Authority.
3. India’s National Disaster Management Plan has been aligned broadly with the goals and
priorities set out in the Sendai Framework for Disaster Risk Reduction.

Which of the statements given above is/are correct?


A. 1 and 2 only
B. 2 and 3 only
C. 1 and 3 only
D. 1, 2 and 3

Answer: C

Explanation:
● Statement 1 is correct: The National Disaster Management Authority is a statutory body
constituted under the Disaster Management Act 2005, with the Prime Minister of India as
its Chairman-; a Vice Chairman with the status of Cabinet Minister, and eight members with
the status of Ministers of State. Hence, statement 2 is incorrect.
● The Disaster Management Act also has statutory provisions for the constitution of the
National Disaster Response Force for the purpose of specialized response to natural and
man-made disasters.
● The NDMA is the apex statutory body for disaster management in India. It is under the
control of the Ministry of Home Affairs (MHA).

National Disaster Management Plan


● The National Disaster Management Plan (NDMP) provides a framework and direction to the
government agencies for all phases of disaster management cycle. The plan is prepared by
the National Disaster Management Authority.
● Statement 3 is correct: The NDMP has been aligned broadly with the goals and priorities
set out in the Sendai Framework for Disaster Risk Reduction. For each hazard, the approach
used in this national plan incorporates the four priorities enunciated in the Sendai
Framework into the planning framework for Disaster Risk Reduction under the five
Thematic Areas for Actions:
○ Understanding Risk
○ Inter-Agency Coordination
○ Investing in DRR – Structural Measures
○ Investing in DRR – Non-Structural Measures
○ Capacity Development

9. With reference to the National Population Register (NPR) and population census, consider
the following statements.
1. While Census is carried out under the Census Act, 1948, NPR is conducted under the
Foreigners Act, 1946.
2. Both the NPR and Census completely exclude foreigners staying in India.

Plot No: 935,6th Avenue, Anna Nagar, Chennai-40


Ph: 044-40483555,9677120226,9677174226
Web: www.officersiasacademy.com
OFFICERS IAS ACADEMY
(IAS Academy by IAS Officers)
3. Both the NPR and Census are conducted by the Office of the Registrar General.

Which of the statements given above is/are correct?


A. 1 only
B. 1 and 2 only
C. 3 only
D. 1 and 3 only

Answer: C

Explanation

What is the National Population Register?


● The National Population Register (NPR) is a Register of usual residents of the country.
● It is being prepared at the local (Village/sub-Town), sub-District, District, State and National
level under provisions of the Citizenship Act 1955 and the Citizenship Rules, 2003.
● It is mandatory for every usual resident of India to register in the NPR.
● A usual resident is defined for the purposes of NPR as a person who has resided in a local
area for the past 6 months or more or a person who intends to reside in that area for the
next 6 months or more.
● NPR cannot be seen as a citizenship registration drive since it would include, for instance,
any foreigner residing in a given locality for over six months.

Present Status
● The data for the NPR is generated through house-to-house enumeration during the “house-
listing” phase of the census, which is held once in 10 years.
● It was collected in 2010 alongwith the houselisting phase of Census of India 2011. The
update of this data was done during 2015 by conducting a door to door survey.
● Now it has been decided to update the NPR along with the Houselisting phase of Census
2021 during April to September 2020 in all the States/UTs except Assam.
● Assam has been excluded because the National Register of Citizens (NRC) exercise has
already been conducted in the state.
● Statement 3 is correct: Both the NPR and Census are conducted by the Office of the
Registrar General under the Ministry of Home Affairs.

What kind of data will NPR collect?


● NPR will collect both demographic data and biometric data. For biometric data it will
depend on Aadhaar, for which it will seek Aadhaar details of the residents.

Why does the government want so much data?


● The objective of the NPR is to create a comprehensive identity database of every usual
resident in the country. It will help the government formulate its policies better and also aid
national security.
● It will streamline data of residents across various platforms and ease the life of those
residing in India by cutting red tape.

Plot No: 935,6th Avenue, Anna Nagar, Chennai-40


Ph: 044-40483555,9677120226,9677174226
Web: www.officersiasacademy.com
OFFICERS IAS ACADEMY
(IAS Academy by IAS Officers)
How NPR differs from population census?
● The decennial census is the largest single source of a variety of statistical information
on different characteristics of all persons in the country. The process began for the first time
in 1872 when India was still under colonial rule.
● The Census is carried out under the Census Act, 1948.
● As opposed to this, the exercise of creating and maintaining a NPR began in the year 2010 to
create a registry of usual residents of the country. NPR is conducted under the Citizenship
Act 1955 and the Citizenship Rules, 2003. Hence, statement 1 is incorrect.
● The Census data does not aim to collect information about individuals but to give an overall
picture of the status or condition of residents of India and the overall population trends.
● The purpose of collecting and analyzing Census data is that it informs planning and policy,
and helps in assessing the impact of existing government policies.
● While the Census Act makes it compulsory for the government to keep the data collected
during census confidential and anonymous, there is no such requirement of confidentiality
for NPR data under the citizenship rules. The NPR is a register that will be in the public
domain with data of individuals.

Are foreigners included?


● Statement 2 is incorrect: Both the NPR and Census include foreigners staying in India —
in NPR's case, those staying for more than six months while in the case of Census, those who
are expected to stay within the geographical limits of this country throughout the
enumeration period are included.

Why in News?
● The Union Cabinet approved a proposal to update the National Population Register.

10. With reference to the Deposit Insurance and Credit Guarantee Corporation (DICGC),
consider the following statements:
1. DICGC is a wholly owned subsidiary of the RBI.
2. In India, the bank deposits of maximum Rs. 1 lakh are covered under the insurance scheme
provided by the DICGC.
3. Deposit insurance facility of DICGC covers deposits of NBFCs and Regional Rural Banks.

Which of the statements given above are correct?


A. 1 and 2 only
B. 1 and 3 only
C. 1 only
D. 2 and 3

Answer: C

Explanation:
● Statement 1 is correct: DICGC is a wholly owned subsidiary of the RBI, created by an act of
parliament in 1961.
● The idea behind the DICGC is to boost the faith of the public in the banking system, and
provide protection against the loss of deposits to a significant extent.

Plot No: 935,6th Avenue, Anna Nagar, Chennai-40


Ph: 044-40483555,9677120226,9677174226
Web: www.officersiasacademy.com
OFFICERS IAS ACADEMY
(IAS Academy by IAS Officers)
● Statement 2 is incorrect: In February 2020, the RBI increased the DICGC insurance
cover on bank deposits from Rs 1 lakh to Rs 5 lakh.
● Statement 3 is incorrect: Banks covered by Deposit Insurance Scheme- All commercial
banks, Local Area Banks, Regional Rural Banks and Co-operative Banks.
● Deposit insurance facility of DICGC is not available to depositors of NBFCs, unlike in the case
of banks.

Why in News?
● Deposit Insurance and Credit Guarantee Corporation (DICGC) permitted to increase Deposit
Insurance Coverage to Rs. 5 lakh from Rs.1 lakh per depositor to provide a greater measure
of protection to depositors.

11. With reference to the International Criminal Court (ICC), consider the following
statements.
1. It is the principal judicial organ of the UN.
2. ICC can try individuals charged with the gravest crimes of concern to the international
community.
3. Most of the members of the UN including India are parties to ICC.

Which of the statements given above is/are correct?


A. 1 and 2 only
B. 2 only
C. 2 and 3 only
D. 1 and 3 only

Answer: B

Explanation:
● The International Criminal Court (ICC) is a permanent judicial body established by the
Rome Statute of the International Criminal Court (1998).
● Statement 1 is incorrect: It is an independent judicial body distinct from the UN.
● Statement 2 is correct: ICC investigates and tries individuals charged with the gravest
crimes of concern to the international community: genocide, war crimes, crimes against
humanity and the crime of aggression.
● The ICC was established as a court of last resort to prosecute the most heinous offenses in
cases where national courts fail to act.
● It is headquartered in The Hague, Netherlands.

Membership
● Statement 3 is incorrect: Only those who ratified the Rome Statute are parties to ICC.
● 122 countries are States Parties to the Rome Statute of the ICC.
● Prominent countries that are not members include China, India, Iraq, Libya, Yemen, Qatar
and Israel.
● The USA and Russia had signed the treaty but did not ratify it.

Plot No: 935,6th Avenue, Anna Nagar, Chennai-40


Ph: 044-40483555,9677120226,9677174226
Web: www.officersiasacademy.com
OFFICERS IAS ACADEMY
(IAS Academy by IAS Officers)
Comparison of ICC and ICJ
S.No International Criminal Court International Court of Justice (ICJ)
(ICC)

1 ▪ ▪I Pri
ndependent judicial body distinct ncipal judicial organ of the UN
from the UN

2 ▪ ▪H Hea
andles prosecutions of individuals rs disputes between sovereign states

3 ▪ ▪E Est
stablished by the Rome Statute in ablished in 1945 by the San Francisco
2002 Conference, which also created the UN

4 ▪ ▪O All
nly those who ratified the Rome members of the UN are parties to the statute
Statute are parties to ICC of the ICJ, and nonmembers may also become
parties

5 ▪ ▪T The
he judges are elected by the judges are elected by the UN General
Assembly of States Parties, the Assembly and the Security Council
court's governing body.

6 ▪ Headquarters of both the courts are


situated at The Hague, Netherlands.

12. With reference to the Blue Dot Network, consider the following statements.
1. It aims to bring governments, the private sector and civil society together to promote high
quality, trusted standards for global infrastructure development.
2. It is envisaged as an extension of the Belt and Road Initiative.
3. The Blue Dot Network includes the US, Japan and Australia.

Which of the statements given above is/are correct?


A. 1 and 2 only
B. 2 and 3 only
C. 1 and 3 only
D. 1, 2 and 3

Answer: C

Explanation:
 Statement 3 is correct: The Blue Dot network (BDN) was jointly launched by the US, Japan and
Australia in 2019 in Thailand.
 Statement 1 is correct: It is meant to be a multi-stakeholder initiative that aims to bring
governments, the private sector and civil society together to promote high quality, trusted

Plot No: 935,6th Avenue, Anna Nagar, Chennai-40


Ph: 044-40483555,9677120226,9677174226
Web: www.officersiasacademy.com
OFFICERS IAS ACADEMY
(IAS Academy by IAS Officers)
standards for global infrastructure development, with a special emphasis on the Indo-
Pacific region.

Significance of BDN
 Statement 2 is incorrect: BDN is seen as a part of the US’s Indo-Pacific strategy, which is aimed
at countering China’s Belt and Road Initiative (BRI).
 However, unlike the BRI, the BDN would not offer public funds or loans for projects.
 The BDN network seeks to set up a globally recognised certification system for infrastructure
projects based on different parameters such as viable funding arrangements, environmental
soundness and high labour standards, so as to shore up quality infrastructural investments.
 India is not a member of both the BDN and China’s BRI.

Why in News?
 Some US officials conveyed their hope that India would join the U.S.-led Blue Dot network.
13. Recently, India and the US concluded the Industrial Security Annex agreement (ISA).
Which among the following statements best describes ISA?
A. It permits the exchange of unclassified geospatial products, topographical, nautical, and
aeronautical data, products and services between India and the US National Geospatial-
Intelligence Agency.
B. It will allow India to procure and transfer specialised equipment for encrypted
communications for US origin military platforms.
C. It will facilitate the exchange of classified military information between private entities of
the two countries.
D. The agreement enables the sharing of military intelligence between the two countries and
requires each country to protect the others' classified information.

Answer: C

Explanation:
● At the second India-US 2+2 dialogue held in 2019, India and the US concluded the Industrial
Security Annex agreement (ISA).
● ISA will facilitate the exchange of classified military information between private
entities of the US and India.
● ISA is expected to deepen industry collaboration, enable release of more advanced
technology and information to India and help make it part of the global supply chain in the
defence sector.
● Option A is related to Basic Exchange and Cooperation Agreement (BECA)
● Option B is related to Communications Compatibility and Security Agreement (COMCASA)
● Option D is related to General Security Of Military Information Agreement (GSOMIA)

GSOMIA
▪ The first of the four foundational agreements of the U.S., the General Security Of Military
Information Agreement (GSOMIA) was signed by India in 2002.
▪ The agreement enables the sharing of military intelligence between the two countries
and requires each country to protect the others' classified information.

Plot No: 935,6th Avenue, Anna Nagar, Chennai-40


Ph: 044-40483555,9677120226,9677174226
Web: www.officersiasacademy.com
OFFICERS IAS ACADEMY
(IAS Academy by IAS Officers)

LEMOA
▪ The second agreement, the Logistics Exchange Memorandum of Agreement (LEMOA),
was signed by the two countries in 2016.
▪ The pact gives both countries access to designated military facilities on either side for
the purpose of refuelling and replenishment in primarily four areas — port calls, joint exercises,
training and humanitarian assistance and disaster relief.
▪ The agreement does not make the provision of logistical support binding on either country,
and requires individual clearance for each request.
▪ This is purely a logistical agreement. There will be no basing of the U.S. troops or assets
on Indian soil.

COMCASA
▪ The third agreement, Communications Compatibility and Security Agreement
(COMCASA) was signed during the inaugural 2+2 dialogue in 2018.
▪ It is an India-specific variant of Communications and Information Security Memorandum of
Agreement (CISMOA).
▪ It enables the two countries to share secure communication and exchange information
on approved equipment during bilateral and multinational training exercises and operations.
▪ It is valid for a period of 10 years from the date of signing.
▪ COMCASA allows India to procure and transfer specialised equipment for encrypted
communications for US origin military platforms like the C-17, C-130 and P-8Is.

BECA
▪ The fourth agreement, Basic Exchange and Cooperation Agreement (BECA) has not yet
been signed.
▪ It permits the exchange of unclassified and controlled unclassified geospatial
products, topographical, nautical, and aeronautical data, products and services between
India and the US National Geospatial-Intelligence Agency (NGA).
▪ BECA is an important enabler of unmanned aerial vehicles from the US, such as the
Predator-B, that use spatial data for accurate strikes on enemy targets.

14. With reference to the newly created Chief of Defence Staff (CDS) post, which one of the
following statements is not correct?
A. CDS will act as the sole military adviser to defence minister on all matters related to defence
B. CDS will act as the permanent chairman of the Chiefs of Staff Committee
C. CDS will head the newly created Department of Military Affairs
D. CDS will not exercise any military command including over three service chiefs

Answer: A

Explanation:
▪ Statement A is incorrect: CDS will be a four star officer and will also act as the principal
military adviser to the defence minister on all tri-service matters. But the three Chiefs will
continue to advise the defence minister on matters exclusively concerning their respective
services.

Plot No: 935,6th Avenue, Anna Nagar, Chennai-40


Ph: 044-40483555,9677120226,9677174226
Web: www.officersiasacademy.com
OFFICERS IAS ACADEMY
(IAS Academy by IAS Officers)
▪ Statements B and C are correct: In addition to heading the newly created
Department of Military Affairs (DMA), the CDS will also be the permanent chairman of the
Chiefs of Staff Committee. The three service chiefs will be members of the Chiefs of Staff
Committee.
▪ Statement D is correct: However CDS will not exercise any military command including
over three service chiefs.

Role and responsibilities of CDS:


▪ As the head of the DMA, the CDS has to facilitate restructuring of military commands for
optimal utilisation of resources by bringing about jointness in operations, including through
establishment of joint/ theatre commands.
▪ The key focus of the CDS office will be involved in procurement of military equipment for
all the three armed forces.
▪ CDS will be member of Defence Acquisition Council and Defence Planning Committee.
▪ Implement Five-Year Defence Capital Acquisition Plan (DCAP), and Two-Year roll-on Annual
Acquisition Plans (AAP), as a follow-up of Integrated Capability Development Plan (ICDP).
▪ Assign inter-Services prioritisation to capital acquisition proposals based on the anticipated
budget.
▪ Prepare strategy papers on military matters for consideration of the competent authority.
15. Consider the following statements about the Indian Ocean Commission (IOC)
1. It is an intergovernmental body that coordinates maritime governance in the south-western
Indian Ocean.
2. Recently, India has been accepted as a member of the Indian Ocean Commission.

Which of the statements given above is/are correct?


A. 1 only
B. 2 only
C. Both 1 and 2
D. Neither 1 nor 2

Answer: A

Explanation:
● Statement 1 is correct: The Indian Ocean Commission (IOC) is an intergovernmental body
created in 1984 that coordinates maritime governance in the south-western Indian Ocean
● It consists of Madagascar, Comoros, French Reunion, Mauritius and Seychelles.
● IOC’s principal mission is to strengthen the ties of friendship between the countries and be
a platform of solidarity for the entire population of the Indianoceanic region.

● The Indian Ocean Commission (IOC) was founded in 1982 and became a regional institution
in 1984.
● It is an intergovernmental organisation consisting of five Member States: The Union of the
Comoros, France Reunion Island, Madagascar, Mauritius and the Seychelles.
● The IOC is the only regional organisation in Africa to represent a group of islands. It defends
the interests of its Member States in Africa and internationally.

Plot No: 935,6th Avenue, Anna Nagar, Chennai-40


Ph: 044-40483555,9677120226,9677174226
Web: www.officersiasacademy.com
OFFICERS IAS ACADEMY
(IAS Academy by IAS Officers)
● The IOC’s mission is to unite Member States’ forces and pool their resources, raise
awareness of the special challenges that developing islands face and promote
Indianoceania as a region of unique human, cultural and natural diversity.

Why in News?
● Statement 2 is incorrect: Recently, India has been accepted as an observer of the Indian
Ocean Commission.

16. Consider the following statements about Asiatic cheetah.


1. It is classified as a “critically endangered” species by the IUCN Red List.
2. At present, they are naturally found only in the Kuno-Palpur Wildlife Sanctuary in India.

Which of the statements given above is/are correct?


A. 1 only
B. 2 only
C. Both 1 and 2
D. Neither 1 nor 2

Answer: A

Explanation:
● Statement 1 is correct: Asiatic cheetah is classified as a “critically endangered” species by
the IUCN Red List, and is believed to survive only in Iran.
● Asiatic cheetahs were once widespread across India but were eradicated in the country as they
were hunted for sport.
● Statement 2 is incorrect: In 1952, Asiatic cheetah was declared extinct from India.

Why in News?
● The Supreme Court lifted its stay on a proposal to introduce African cheetahs from Namibia into
the Indian habitat on an experimental basis.
● Kuno-Palpur Wildlife Sanctuary in Madhya Pradesh is most likely to be selected as the new
home for these mammals.

17. Consider the following statements


1. "Orphan drugs" are medicinal products intended for diagnosis, prevention or treatment of
life-threatening or very serious diseases or disorders that are rare.
2. When a drug is included in Schedule H the Drugs and Cosmetic Rules of 1945, it is
prohibited from sales anywhere in India.

Which of the statements given above is/are incorrect?


A. 1 only
B. 2 only
C. Both 1 and 2
D. Neither 1 nor 2

Answer: B

Plot No: 935,6th Avenue, Anna Nagar, Chennai-40


Ph: 044-40483555,9677120226,9677174226
Web: www.officersiasacademy.com
OFFICERS IAS ACADEMY
(IAS Academy by IAS Officers)
Explanation:
● Statement 1 is correct: "Orphan drugs" are medicinal products intended for
diagnosis, prevention or treatment of life-threatening or very serious diseases or disorders
that are rare.
● These drugs are called “orphan” because under normal market conditions the
pharmaceutical industry has little interest in developing and marketing products intended
for only a small number of patients.
● For drug companies, the extremely high cost of bringing a medicinal product to market
would not be recovered by the expected sales of the product.
● The Drugs and Cosmetic Rules of 1945 contains provisions for the classification of the drugs
under various given schedules including details regarding the storage, display, sale and the
prescription of an individual schedule.
● Statement 2 is incorrect: When a drug is included in Schedule H the Drugs and Cosmetic
Rules of 1945, it can be sold only against the prescription of a registered medical
practitioner.

18. Which one of the following is not a Strategic Petroleum Reserve site in India?
A. Visakhapatnam
B. Mangaluru
C. Kandla
D. Padur

Answer: C

Explanation:
● Strategic Petroleum Reserves (SPRS) are huge stockpiles of crude oil established to tackle
emergency situations. SPRs significantly help India's energy security and will insulate the
country from external price and supply shocks.
● Currently, India has 5.33 Million Metric Tonnes (MMT) of SPR capacity across three
locations — 1.3 MMT at Visakhapatnam, 1.5 MMT at Mangaluru and 2.5 mt at Padur
(Karnataka)
● At full capacity of 5.33 million tonnes, these can provide fuel security for nine days for the
country.
● In addition, the government is planning to create 12.5 MT storage capacity at Padur,
Chandikhol (Odisha), Bikaner (Rajasthan) and Rajkot (Gujarat).
● The government created a special purpose vehicle (SPV)—Indian Strategic Petroleum
Reserves Ltd. (ISPRL)—-for building these SPRs.

19. The GAVI Alliance sometimes seen in the news in the context of
A. Leading international non-profit and non-governmental organization that safeguards the
right to freedom of information.
B. Wildlife Trade Monitoring Network working to ensure that trade in wild plants and animals
is not a threat to the conservation of nature.
C. Global health partnership of public and private sector organizations dedicated to
immunisation for children in the world’s poorest countries.
D. Global alliance focused on enhancing the cyber security readiness and response of public
and private sector entities.

Plot No: 935,6th Avenue, Anna Nagar, Chennai-40


Ph: 044-40483555,9677120226,9677174226
Web: www.officersiasacademy.com
OFFICERS IAS ACADEMY
(IAS Academy by IAS Officers)

Answer: C

Explanation:
About GAVI Alliance
● The GAVI Alliance (formerly the Global Alliance for Vaccines and Immunisation) is a global
health partnership of public and private sector organizations dedicated towards creating
equal access to new and underused vaccines for children living in the world’s poorest
countries.
● GAVI’s strategy supports its mission to save children’s lives and protect people’s health by
increasing access to immunisation in poor countries.
● Its partners provide funding for vaccines and intellectual resources for care advancement.
They contribute, also, to strengthening the capacity of the health system to deliver
immunisation and other health services in a sustainable manner.

Why in News?
● Prime Minister Narendra Modi participated in the virtual Global Vaccine Summit hosted
by UK Prime Minister Boris Johnson and pledged $15 million to Gavi alliance.
● Option A is related to Reporters Without Borders.
● Option B is related to TRAFFIC.

20. The main objective of the PM SVANidhi Scheme is


A. Aid, assist and advice farmers on how to improve their farming techniques and raise their
incomes.
B. Providing affordable loans to street vendors.
C. Providing health insurance to COVID-19 frontline workers.
D. Implementing Direct Benefit Transfer for procurement of food grains by the Food
Corporation of India.

Answer: B

Explanation:
● The Central government launched the Pradhan Mantri Street Vendor’s Atmanirbhar Nidhi
(PM SVANidhi) Scheme on 1 June 2020 for providing affordable loans to street vendors.
● The vendors can avail a working capital loan of up to Rs. 10,000, which is repayable in
monthly instalments within a year. The loans would be without collateral.
● The main objective of the scheme is to ensure that daily wage earners like vegetable sellers,
fruit sellers, earn their livelihood. This short term assistance of Rs. 10,000 will enable small
street vendors to restart their work which is badly hit due to Coronavirus (COVID-19) lock-
down.

21. With reference to the Members of Parliament Local Area Development Scheme
(MPLADS), consider the following statements.
1. MPs are entitled to recommend works to the tune of Rs 2 crore annually

Plot No: 935,6th Avenue, Anna Nagar, Chennai-40


Ph: 044-40483555,9677120226,9677174226
Web: www.officersiasacademy.com
OFFICERS IAS ACADEMY
(IAS Academy by IAS Officers)
2. MPs shall recommend works costing at least 15 per cent of the MPLADS entitlement
for the year for areas inhabited by Scheduled Caste population and 7.5 per cent for
areas inhabited by S.T. population
3. Nominated Members of Rajya Sabha can recommend works anywhere in the country

Which of the statements given above is/are correct?


A. 1 and 2 only
B. 2 and 3 only
C. 1 and 3 only
D. 1, 2 and 3

Answer: B

Explanation
● The Members of Parliament Local Area Development Scheme (MPLADS) is a programme
first launched 1993, aimed towards providing funds for developmental works
recommended by individual MPs.
● The objective of the scheme is to enable MPs to recommend works of developmental nature
with emphasis on the creation of durable community assets based on the locally felt needs
to be taken up in their Constituencies.
● Statement 1 is incorrect: Under the MPLADS scheme, each MP has the choice of suggesting
to the District Collector works to the tune of Rs 5 crore per annum to be taken up in his or
her constituency.
● Statement 3 is correct: Lok Sabha Members can recommend works within their
Constituencies and Elected Members of Rajya Sabha can recommend works within the State
of Election (with select exceptions). Nominated Members of Rajya Sabha can recommend
works anywhere in the country.
● Under MPLADS, the role of the MPs is limited only upto recommendation of works.
Thereafter, it is the responsibility of the District Authority to sanction, execute and
complete the works recommended by Members of Parliament within the stipulated time.
● Statement 2 is correct: The guidelines recommend MPs to suggest works costing at least
15 per cent of their MPLADS entitlement for the year for areas inhabited by Scheduled Caste
population and 7.5 per cent for areas inhabited by ST population.
● In case there is insufficient tribal population in the area of Lok Sabha Member, they may
recommend this amount for the creation of community assets in tribal areas outside of their
constituency but within their State of election.

What kind of projects are executed?


● The guidelines lay down a number of development works, including construction of railway
halt stations, providing financial assistance to recognised educational bodies, cooperative
societies, bar associations, installing CCTV cameras, and rainwater harvesting systems,
● The MPLADS funds can be merged with other schemes such as MGNREGA and Khelo India.
● MPLADS was held constitutionally valid by the Supreme Court in its May 6, 2010 judgment.

Why in News?

Plot No: 935,6th Avenue, Anna Nagar, Chennai-40


Ph: 044-40483555,9677120226,9677174226
Web: www.officersiasacademy.com
OFFICERS IAS ACADEMY
(IAS Academy by IAS Officers)
● Recently, the Union Cabinet gave its nod to the temporary suspension of MPLAD
Funds during 2020-21 and 2021-22 in view of the adverse impact of the outbreak of
COVID-19 in India.
● The consolidated amount of MPLAD Funds for 2 years – Rs 7,900 crores – will go to the
Consolidated Fund of India.

22. With reference to the Pradhan Mantri Ujjwala Yojana, consider the following statements.
1. It is implemented by the Ministry of Petroleum & Natural Gas.
2. Beneficiaries under the scheme is restricted to Socio Economic and Caste Census identified
households.
3. Under the scheme, cash assistance of Rs. 1600 is given to the beneficiaries to get a deposit-
free new LPG connection.

Which of the statements given above is/are correct?


A. 1 and 2 only
B. 2 and 3 only
C. 1 and 3 only
D. 1, 2 and 3

Answer: C

Explanation:
● Statement 1 is correct: Launched in 2016, Pradhan Mantri Ujjwala Yojana is a scheme of
the Ministry of Petroleum & Natural Gas which aims to safeguard the health of women &
children by providing them with a clean cooking fuel – LPG.
● The scheme provides free LPG connections to economically weaker families. The
connections are issued in the name of women of the households.
● Statement 3 is correct: Under the scheme, cash assistance of Rs. 1600 is given to the
beneficiaries to get a deposit-free new connection.
● Initially, the beneficiaries under PMUY were identified either from the SECC list or from
seven other identified categories.
● Statement 2 is incorrect: In 2018, the Government extended the benefit to all left out
poor families not having LPG connections and not covered under the existing beneficiary
categories.

Why in News?
● Recently, Himachal Pradesh became the first state in the country to have 100% LPG gas
coverage.
● Following the implementation of the Pradhan Mantri Ujjwala Yojana, the state government
launched its own Himachal Grihini Suvidha Yojana in 2018 to cover the remaining
households.

23. The main objective of the GOAL programme is


A. To provide an alternative source of livelihoods to members of Self-Help Groups.
B. To provide mentorship to tribal youth through digital mode.
C. Department of Science & Technology’s initiative to accelerate research in Quantum
computing.

Plot No: 935,6th Avenue, Anna Nagar, Chennai-40


Ph: 044-40483555,9677120226,9677174226
Web: www.officersiasacademy.com
OFFICERS IAS ACADEMY
(IAS Academy by IAS Officers)
D. Creating awareness and understanding of the value of Snow Leopard for the
ecosystem.

Answer: B

Explanation:
● The Ministry of Tribal Affairs has launched the “GOAL (Going Online As Leaders)”
programme in partnership with Facebook.
● The GOAL programme is designed to provide mentorship to tribal youth through digital
mode and reduce the gap between tribal and non-tribal youth.
● In this program, 5000 scheduled tribe youth will get training by experts from different
disciplines and fields.

● Option A is related to Aajeevika Grameen Express Yojana (AGEY).


● Option C is related to the Quantum-Enabled Science & Technology (QuEST) programme.
● Option D is related to the Global Snow Leopard & Ecosystem Protection (GSLEP)
programme.

24. ‘Artemis Accords’ is sometimes seen in the news in the context of


A. A common set of principles covering how the Moon is to be explored and its resources
utilized.
B. To ensure fair and equitable sharing of benefits arising from the utilization of genetic
resources.
C. Global treaty which proposes to ban the development, production, and stockpiling of an
entire category of weapons of mass destruction.
D. To inspire, celebrate and advance the critical role that education has in delivering the
Sustainable Development Goals (SDGs).

Answer: A

Explanation:
● NASA has formally announced the "Artemis Accords", a series of principles and processes
whereby America and other countries would agree to a common set of principles covering
how the Moon is to be explored and its resources utilized.
● The accords are named after NASA’s Artemis programme. Via the Artemis program, NASA
will land the first woman and the next man on the Moon by 2024.
● Artemis accord promotes transparency and communication between nations, requiring
signatories to share their lunar plans, register any spacecraft sent to or around the moon
and release scientific data to the public.
● Under the Accord, nations will be able to set “safety zones” to protect their activities on the
moon, they will have to work to mitigate the effects of debris in orbit around the moon and
they will agree to provide emergency assistance to any astronauts in distress.

● Option B is related to Nagoya Protocol.


● Option D is related to SDG Accord.

25. Which of the following institutions are excluded from the application of the RTI Act?

Plot No: 935,6th Avenue, Anna Nagar, Chennai-40


Ph: 044-40483555,9677120226,9677174226
Web: www.officersiasacademy.com
OFFICERS IAS ACADEMY
(IAS Academy by IAS Officers)
1. Non-Government organization substantially financed by the appropriate Government
2. Intelligence Bureau
3. The Office of the Chief Justice of India

Select the correct answer using the codes given below


A. 1 and 3 only
B. 2 only
C. 2 and 3 only
D. 1, 2 and 3

Answer: B

Explanation:

Public authority under RTI Act


● Public authorities are the repository of information which the citizens have the right to have
under the Right to Information Act (RTI Act).
● The RTI Act defines “public authorities” in Section 2(h).
● A “public authority” means any authority or body or institution of self- government
established or constituted –
o by or under the Constitution;
o by any other law made by Parliament;
o by any other law made by State Legislature;
o by notification issued or order made by the appropriate Government, and includes any
▪ body owned, controlled or substantially financed;
▪ Non-Government organization substantially financed, directly or indirectly
by funds provided by the appropriate Government.

● Second schedule of the RTI Act has a list of organizations which are exempt from the
application of this Act. The organizations are as follows:-
○ Intelligence Bureau
○ Research and Analysis Wing of the Cabinet Secretariat
○ Directorate of Revenue Intelligence
○ Central Economic Intelligence Bureau
○ Directorate of Enforcement
○ Narcotics Control Bureau
○ Aviation Research Centre
○ Special Frontier Force
○ Border Security Force
○ Central Reserve Police Force
○ Indo-Tibetan Border Police
○ Central Industrial Security Force
○ National Security Guards
○ Assam Rifles
○ Sashastra Seema Bal
○ Directorate General of Income Tax (Investigation)
○ National Technical Research Organization

Plot No: 935,6th Avenue, Anna Nagar, Chennai-40


Ph: 044-40483555,9677120226,9677174226
Web: www.officersiasacademy.com
OFFICERS IAS ACADEMY
(IAS Academy by IAS Officers)
○ Financial Intelligence Unit India
○ Special Protection Group
○ Defense Research and Development Organization
○ Border Road Development Board, and
○ National Security Council Secretariat
● In November 2019, the Supreme court gave a verdict that the office of CJI is a public
authority under the RTI act.

26. With reference to Operation Greens, consider the following statements.


1. It was launched by the Ministry of Food Processing Industries to stabilize the supply of
certain crops and to ensure their availability throughout the country round the year without
price volatility.
2. The scheme is restricted to Tomato, Onion and Potato (TOP) crops.
3. NAFED acts as the Nodal Agency to implement price stabilisation measures.

Which of the statements given above is/are correct?


A. 1 and 2 only
B. 1 and 3 only
C. 2 and 3 only
D. 1, 2 and 3

Answer: B

Explanation:
● Statement 1 is correct: Operation Greens was launched by the Ministry of Food
Processing Industries to stabilize the supply of Tomato, Onion and Potato (TOP) crops and
to ensure availability of TOP crops throughout the country round the year without price
volatility.
● Statement 3 is correct: National Agricultural Cooperative Marketing Federation of
India (NAFED) acts as the nodal agency to implement price stabilisation measures under
Operation Greens.

Objectives
● Enhancing value realisation of TOP farmers by targeted interventions to strengthen TOP
production clusters and their FPOs, and linking/connecting them with the market.
● Price stabilisation for producers and consumers by proper production planning in the TOP
clusters and introduction of dual use varieties.
● Reduction in post-harvest losses by creation of farm gate infrastructure, development of
suitable agro-logistics, creation of appropriate storage capacity linking consumption
centres.
● Increase in food processing capacities and value addition in TOP value chain with firm
linkages with production clusters.
● Setting up of a market intelligence network to collect and collate real time data on demand
and supply and price of TOP crops.

Strategy

Plot No: 935,6th Avenue, Anna Nagar, Chennai-40


Ph: 044-40483555,9677120226,9677174226
Web: www.officersiasacademy.com
OFFICERS IAS ACADEMY
(IAS Academy by IAS Officers)
● The strategy will comprise of a series of measures as decided by the Ministry which
include:
● Short term Price Stabilisation Measures: MoFPI will provide 50% of the subsidy on the
following two components:
○ Transportation of TOP Crops from production to storage;
○ Hiring of appropriate storage facilities for TOP Crops;
● Long Term Integrated value chain development projects
○ Capacity Building of FPOs & their consortium
○ Quality production
○ Post-harvest processing facilities
○ Agri-Logistics
○ Marketing / Consumption Points
○ Creation and Management of e-platform for demand and supply management of TOP
Crops.

● Statement 2 is incorrect: Recently, the Ministry of Food Processing Industries has formally
announced the expansion of Operation Greens scheme from tomato, onion and potato crops
to all perishable fruits and vegetables.

27. With reference to the United Nations Security Council (UNSC), consider the following
statements.
1. There are a total of fifteen members in the UNSC.
2. Decisions of the Security Council on procedural matters shall be made by an affirmative vote
of nine members.
3. Only the five permanent members of the UNSC enjoy the veto power.
4. Currently, India is one of the non-permanent members of the UNSC.

Which of the statements given above is/are correct?


A. 1 and 4 only
B. 1, 2 and 3 only
C. 1, 3 and 4 only
D. 1, 2, 3 and 4

Answer: B

Explanation:
● The United Nations Security Council (UNSC) was established in 1946 as one of the six
principal organs of the UN. It is generally viewed as the apex of the UN system.
● It is responsible for the maintenance of international peace and security.
● Its powers include the establishment of peacekeeping operations, the establishment of
international sanctions, and the authorization of military action through Security Council
resolutions.
● It is the only UN body with the authority to issue binding resolutions to member states.

Membership

Plot No: 935,6th Avenue, Anna Nagar, Chennai-40


Ph: 044-40483555,9677120226,9677174226
Web: www.officersiasacademy.com
OFFICERS IAS ACADEMY
(IAS Academy by IAS Officers)
● Statement 1 is correct: UNSC consists of 15 Members (including 5 permanent
members- (P-5) United States, Russia, China, United Kingdom and France) and each
member has one vote.
● Statement 3 is correct: The UN Charter affords the veto power only to the five
permanent members of the UNSC.
● These permanent members can veto any substantive Security Council resolutions, including
those on the admission of new member states.
● The Security Council also has 10 non-permanent members, elected on a regional basis as
follows:
○ five for African and Asian States;
○ one for Eastern European States;
○ two for the Latin American and Caribbean States; and
○ two for Western European and other States.
● The body's presidency rotates monthly among its members.
● Statement 4 is incorrect: The present composition of ten non-permanent members (with
end of term year):
● Belgium (2020)
● Dominican Republic (2020)
● Estonia (2021)
● Germany (2020)
● Indonesia (2020)
● Niger (2021)
● Saint Vincent and the Grenadines (2021)
● South Africa (2020)
● Tunisia (2021)
● Viet Nam (2021)

● Recently, India was elected as a non-permanent member of the UNSC for a two year term
starting on January 1, 2021 from the Asia-Pacific category after winning 184 votes in the
193-member General Assembly.
● Along with India, Ireland, Mexico and Norway also won the Security Council elections.
● To be elected to the Council, candidate countries need a two-thirds majority of ballots of
Member States that are present and voting in the General Assembly.
● India has already held a non-permanent seat on the UNSC for seven terms.

Vote and Majority Required


● Each member of the Security Council shall have one vote.
● Statement 2 is correct: Decisions of the Security Council on procedural matters shall be
made by an affirmative vote of nine members.
● Decisions of the Security Council on all other matters shall be made by an affirmative vote
of nine members including the concurring votes of the permanent members.
● However, any member, whether permanent or nonpermanent, must abstain from voting in
any decision concerning the peaceful settlement of a dispute to which it is a party.

28. With reference to the Inverted duty structure, consider the following statements.
1. It is a situation where import duty on raw materials is low compared to the import duty on
finished goods.

Plot No: 935,6th Avenue, Anna Nagar, Chennai-40


Ph: 044-40483555,9677120226,9677174226
Web: www.officersiasacademy.com
OFFICERS IAS ACADEMY
(IAS Academy by IAS Officers)
2. Government uses such a tax regime to boost the competitiveness of domestic
industries against imported finished goods.

Which of the statements given above is/are correct?


A. 1 only
B. 2 only
C. Both 1 and 2
D. Neither 1 nor 2

Answer: D

Explanation:
● Statement 1 is incorrect: Inverted duty structure is a situation where import duty on
finished goods is low compared to the import duty on raw materials that are used in the
production of such finished goods.
● For example, suppose the tariff on the import of tyres is 10% and the tariff on the imports of
natural rubber which is used in the production of tyres is 20%; this is a case of inverted
duty structure.
● When the import duty on raw materials is high, it will be more difficult to produce the
concerned good domestically at a competitive price. Several industries depend on imported
raw materials and components.
● High tax on the raw materials compels them to raise prices. On the other hand, foreign
finished goods will be coming at a reduced price because of low tax advantage. In
conclusion, manufactured goods by the domestic industry become uncompetitive against
imported finished goods. Hence, statement 2 is incorrect.
● The disadvantage of the inverted duty structure increases with the increased use of
imported raw materials. An inverted duty structure discourages domestic value addition.

29. With reference to the Data adequacy status, consider the following statements.
1. It is granted by the WTO under the Trade-Related Aspects of Intellectual Property Rights
(TRIPS) Agreement.
2. When a country has been awarded the status, information can pass freely between it and
some foreign territories without further safeguards being required.

Which of the statements given above is/are correct?


A. 1 only
B. 2 only
C. Both 1 and 2
D. Neither 1 nor 2

Answer: B

Explanation:
● Statement 1 is incorrect: Data adequacy is a status granted by the European Union to
countries outside the European Economic Area (EEA) who provide a level of personal data
protection comparable to that provided in European law (General Data Protection
Regulation (GDPR)).

Plot No: 935,6th Avenue, Anna Nagar, Chennai-40


Ph: 044-40483555,9677120226,9677174226
Web: www.officersiasacademy.com
OFFICERS IAS ACADEMY
(IAS Academy by IAS Officers)
● Statement 2 is correct: When a country has been awarded the status, information
can pass freely between it and the EEA without further safeguards being required.
Data adequacy can also be awarded to specified sectors of an economy or international
organisations.
● The European Union has granted data adequacy status to more than 10 countries so far.

Why in News?

● India is planning to seek adequacy status with the European Union.


● This would allow European Union tech firms to store and transfer data in India, and could
enable companies in Europe to outsource data-intensive projects to India.

30. With reference to the new classification of Micro, Small & Medium Enterprises (MSMEs)
in India, consider the following statements.
1. MSMEs will be categorised based only on the investment in machinery or equipment.
2. There will be no distinction between Manufacturing and Service MSMEs

Which of the statements given above is/are correct?


A. 1 only
B. 2 only
C. Both 1 and 2
D. Neither 1 nor 2

Answer: B

Explanation:
● MSMEs contribute 29.7% of GDP and 49.66% of Indian Exports.

● Statement 2 is correct: Recently, the government has changed the basic definition of
MSME and also end the difference between the manufacturing and services sector.
● Statement 1 is incorrect: Till now, MSMEs are categorised based only on the investment in
machinery or equipment. The new classification has raised the investment limit and
included annual turnover as an additional criteria.
○ Accordingly, units having investment less than Rs 1 crore and turnover less than Rs
5 crore will be called Micro units.
○ Investment between Rs 1 and Rs 10 crore and turnover of Rs 5 crore to Rs 50 crore
will be categorised as Small Enterprises.
○ Units having investment between Rs 10 crore but up to Rs 50 crore and turnover
between Rs 50 crore and Rs 250 crore will now be known as Medium Enterprises.
Rationale for the move
● It has been a long-standing demand from industry to hike the investment limits, as with
inflation, units often cross the threshold that will bring them benefits. To prevent this, they
either run their operations at a reduced level or incorporate multiple units so that turnover
is distributed in a way that they remain within the threshold that will give them the
benefits.

Plot No: 935,6th Avenue, Anna Nagar, Chennai-40


Ph: 044-40483555,9677120226,9677174226
Web: www.officersiasacademy.com
OFFICERS IAS ACADEMY
(IAS Academy by IAS Officers)
● With the revised definitions of MSMEs, they will not have to worry about growing
their size and can still avail benefits.

GEOGRAPHY
1. Consider the following statements:
1. At the equator the sun rises and sets almost vertically so the time it takes to pass through
the ‘twilight zone’ will be longer than for temperate latitudes.
2. The brief period between sunrise and full daylight is called twilight.

Which of the above mentioned statements is/are correct?


(a) Only 1
(b) Only 2
(c) Both 1 and 2
(d) Neither 1 nor 2
Ans : D
 The brief period between sunrise and full daylight is called dawn, and that between sunset
and complete darkness is termed twilight. This is caused by the fact that during the periods
of dawn and twilight the earth receives diffused or refracted light from the sun whilst it is
still below the horizon.
Since the sun rises and sets in a vertical path at the equator the period during which refracted light
is received is short. But in temperate latitudes, the sun rises and sets in an oblique path and the
period of refracted light is longer. It is much longer still at the poles, so that the winter darkness is
really only twilight most of the time.

2. Which of these regions of India is most suitable for growing rice for all three cropping seasons –
kharif, rabi and zaid?
a) North-western India
b) Eastern India
c) Southern India
d) Western India

Ans: C
Climate with large number of days with sunshine and fertile soils permit growing of more than one
crop annually on the same plot.
Southern India, being nearer to the equator than other regions of India, has more even temperature
conditions than the rest of India.

Plot No: 935,6th Avenue, Anna Nagar, Chennai-40


Ph: 044-40483555,9677120226,9677174226
Web: www.officersiasacademy.com
OFFICERS IAS ACADEMY
(IAS Academy by IAS Officers)
While rainfall is not evenly distributed throughout the year, irrigation structures allow sowing
of crops in multiple seasons. In fact, rice is sown in all three seasons here. It is both a kharif
and rabi crop despite the huge water demands of rice.

3. Self-ploughing is a phenomenon associated with this soil. They are formed by the denudation of
volcanic rocks. They are deep and impermeable.
Which of the following is the type of soil that above statements attribute to?
a) Black Soil
b) Laterite Soil
c) Peat Soil
d) Arid Soil

Ans: A
The black soil is very deep. These soils are also known as the Regur Soil or the Black Cotton Soil.
The black soils are generally clayey, deep and impermeable. They swell and become sticky when
wet and shrink when dried. So, during the dry season, these soil develop wide cracks. Thus, there
occurs a kind of self-ploughing. Because of this character of slow absorption and loss of moisture,
the black soil retains the moisture for a very long time, which helps the crops, especially, the rain
fed ones, to sustain even during the dry season. Black soils are formed by the denudation of
volcanic rocks. Chemically, the black soils are rich in lime, iron, magnesia and alumina. They also
contain potash. But they lack in phosphorous, nitrogen and organic matter. The colour of the soil
ranges from deep black to grey.

4. Why tropical cyclones affect mainly the coastal areas of India but not the interior parts?
a) Pressure gradient force is minimal in the coastal areas compared to the interiors.
b) High temperature across the coastal areas than interiors.
c) Greater Coriolis force on the coastal land compared to that of interior.
d) Energy driving the cyclone decreases with the distance from the sea.

Ans: D
The energy of the tropical cyclone comes from the latent heat released by the warm moist air.
Hence, with the increase in distance from the sea, the force of the cyclone decreases. So, the coastal
areas are often struck by severe cyclonic storms. Hence, (d) is the correct.

5. Consider the following statements about surface waves:


1. They propagate on the interface of two mediums like earth and atmosphere.
2. They can travel in solid, liquid and gas materials.
3. They are last to report on seismograph and are more destructive.

Which of the statements given above is/are correct?


a) 1 only
b) 1 and 2 only
c) 1 and 3 only

Plot No: 935,6th Avenue, Anna Nagar, Chennai-40


Ph: 044-40483555,9677120226,9677174226
Web: www.officersiasacademy.com
OFFICERS IAS ACADEMY
(IAS Academy by IAS Officers)
d) 2 and 3 only

Ans: C
Earthquake waves are basically of two types — body waves and surface waves. Body waves are
generated due to the release of energy at the focus and move in all directions travelling through the
body of the earth. Hence, the name body waves. The body waves interact with the surface rocks and
generate new set of waves called surface waves. These waves move along the surface on the
interface of earth and atmosphere.
The velocity of waves changes as they travel through materials with different densities. The denser
the material, the higher is the velocity. Their direction also changes as they reflect or refract when
coming across materials with different densities. Surface waves do not travel in interior of earth
like body waves. This characteristic of the S-waves is quite important. It has helped scientists to
understand the structure of the interior of the earth. Reflection causes waves to rebound whereas
refraction makes waves move in different directions. The variations in the direction of waves are
inferred with the help of their record on seismograph.
The surface waves are the last to report on seismograph. These waves are more destructive. They
cause displacement of rocks, and hence, the collapse of structures occurs.

6. Which of the following provide information about the Earth’s interior?


1. Volcanic eruptions
2. Meteors
3. Seismic Activities
4. Deep ocean drilling projects

Select the correct answer using the code given below.


a) 1, 2 and 3 only
b) 2 and 3 only
c) 2, 3 and 4 only
d) 1, 2, 3 and 4

Ans : D
The various sources of information about the Earth's interior can be classified into direct and
indirect sources.
Direct sources include:
•Study of surface rock or the rocks - from mining areas
•Deep Ocean Drilling Projects
•Volcanic eruption
Indirect sources includes analysis of properties of matter indirectly provides information about the
interior. Like,
•Meteors
•Gravitation
•Magnetic field, and
•Seismic activity etc.

Plot No: 935,6th Avenue, Anna Nagar, Chennai-40


Ph: 044-40483555,9677120226,9677174226
Web: www.officersiasacademy.com
OFFICERS IAS ACADEMY
(IAS Academy by IAS Officers)

7. Which the following statements with reference to the Sargasso Sea is not correct?
a) It is named after free-floating seaweed called Sargassum.
b) It is the only sea without a land boundary.
c) It is surrounded by four currents that form an ocean gyre.
d) It lies in South Atlantic Ocean.

Answer: D

The Sargasso Sea lies within the North Atlantic Ocean, surrounded by four currents that form an
ocean gyre. The Sargasso Sea is bounded by the Gulf Stream on the west, the North Atlantic Current
on the north, the Canary Current on the east, and the North Equatorial Current on the south. It has
been named for a genus of free-floating seaweed called Sargassum. It has no land boundaries and is
defined by ocean currents, western boundary by Gulf Stream, the north by the North Atlantic
Current, the east by the Canary Current, and the south by the North Atlantic Equatorial Current.

8. Consider the following statements.


1. Western Coast is known as the coast of emergence.
2. The broader continental shelf in the eastern coast makes it unviable for the formation of
natural ports.
3. A water body named 8-degree channel separates Andaman from Nicobar.
4. The entire island group of Lakshadweep is made up of coral deposits.

Plot No: 935,6th Avenue, Anna Nagar, Chennai-40


Ph: 044-40483555,9677120226,9677174226
Web: www.officersiasacademy.com
OFFICERS IAS ACADEMY
(IAS Academy by IAS Officers)
Select the correct statement (s) using the codes given below.
a) 1 and 3 only
b) 2 and 4 only
c) 1, 2 and 4 only
d) 2, 3 and 4 only

Ans : B
Western coast is a coast of submergence except for the Malabar Coast where it is emergent in
nature. Hence, statement 1 is wrong.
Eastern coastal shelf runs about 500km in the sea and is broader than western coastal shelf. Due to
the emergent nature of eastern coast it is less feasible for development for natural ports.
Water body named ten degree channel separates Andaman from Nicobar Islands. Hence statement
3 is incorrect.
Entire group of Lakshadweep islands is built of coral deposits.

9. The terms Roaring Forties, Furious Fifties and Shrieking Sixties are used in relation to
(a) Ocean currents
(b) Jet streams
(c) Easterly winds
(d) Westerly winds

Ans : D

From the Sub-Tropical High-Pressure Belts, winds blow towards the Temperate Low-Pressure Belts
as the variable Westerlies. Under the effect of the Coriolis Force, they become the South-Westerlies
in the northern hemisphere and the North-Westerlies in the southern hemisphere.
They are more variable in the northern hemisphere. but they play a valuable role in carrying warm
equatorial waters and winds to western coasts of temperate lands. This warming effect and other
local pressure differences have resulted in a very variable climate in the temperate zones,
dominated by the movements of cyclones and anticyclones.
In the southern hemisphere where there is a large expanse of ocean, 40 degrees S to 60 degree S,
Westerlies blow with much greater force and regularity throughout the year. They bring much
precipitation to the western coasts of continents. The weather is damp and stormy. It is thus usual
for seafarers to refer to the Westerlies as the Roaring Forties, Furious Fifties and Shrieking Sixties,
according to the varying degree of storminess in the latitudes which they blow.

10. These are the most widespread forests in India. They spread over regions which receive rainfall
between 70-200 cm. On the basis of the availability of water, these forests are further divided into
moist and dry. Teak, sal, shisham, Tendu, palas, and sandalwood are the main species of these
forests.
The above passage describes which of the following?
(a) Semi Evergreen Forests

Plot No: 935,6th Avenue, Anna Nagar, Chennai-40


Ph: 044-40483555,9677120226,9677174226
Web: www.officersiasacademy.com
OFFICERS IAS ACADEMY
(IAS Academy by IAS Officers)
(b) Tropical Deciduous Forest
(c) Thorn Forest
(d) Montane Forest

Ans: B

Deciduous Forests are the most widespread forests in India. They are also called the monsoon
forests. They spread over regions which receive rainfall between 70-200 cm. On the basis of the
availability of water, these forests are further divided into moist and dry deciduous.
The moist deciduous forests are more pronounced in the regions which record rainfall between
100-200 cm. These forests are found in the northeastern states along the foothills of Himalayas,
eastern slopes of the Western Ghats and Orissa. Teak, sal, shisham, hurra, mahua, amla, semul,
kusum, and sandalwood etc. are the main species of these forests.
Dry deciduous forest covers vast areas of the country, where rainfall ranges between 70 -100 cm.
On the wetter margins, it has a transition to the moist deciduous, while on the drier margins to
thorn forests. These forests are found in rainier areas of the Peninsula and the plains of Uttar
Pradesh and Bihar. As the dry season begins, the trees shed their leaves completely and the forest
appears like a vast grassland.

11. In the context of the formation of soil consider the following statements:
1. Climatic factors influence the rate of weathering.
2. Flora, fauna, and micro-organisms affect the rate of humus formation.
3. Parent rocks determine the texture and permeability of soil.
Which of the statements given above is/are correct?
(a) 1 only
(b) 1 and 2 only
(c) 2 and 3 only
(d) 1, 2 and 3
Ans : D

The major factors of soil formation are the nature of the parent rock and climatic factors. Other
factors are the topography, role of organic material and time taken for the composition of soil
formation. All these differ from place to place.
Parent Rock: Determines colour, texture, chemical properties mineral, content, permeability
Climate: Temperature, Rainfall influence rate of weathering and humus formation
Time: Determines the thickness of soil profile
Flora, Fauna and Micro-organism: Affect the rate of humus formation

12. Consider the following statements with reference to the structure of the atmosphere:
1. Thickness of the troposphere is greatest at the equator.
2. The air temperature at the tropopause over the equator is less than that at the poles.

Plot No: 935,6th Avenue, Anna Nagar, Chennai-40


Ph: 044-40483555,9677120226,9677174226
Web: www.officersiasacademy.com
OFFICERS IAS ACADEMY
(IAS Academy by IAS Officers)
3. The temperature within the mesosphere increases with the increase in altitude.
Which of the statements given above is/are correct?
(a) 1 and 2 only
(b) 3 only
(c) 1 only
(d) 1, 2 and 3
Ans A
The column of atmosphere is divided into five different layers depending upon the temperature
condition. They are: troposphere, stratosphere, mesosphere, thermosphere and exosphere.

Statement 1 is correct: The troposphere is the lowermost layer of the atmosphere. Its average
height is 13 km and extends roughly to a height of 8 km near the poles and about 18 km at the
equator. Thickness of the troposphere is greatest at the equator because heat is transported to
great heights by strong convectional currents. This layer contains dust particles and water vapour.
All changes in climate and weather take place in this layer. The temperature in this layer decreases
at the rate of 1°C for every 165m of height. This is the most important layer for all biological
activity.

Statement 2 is correct: The zone separating the tropsophere from stratosphere is known as the
tropopause. The air temperature at the tropopause is about minus 800 degree C over the equator
and about minus 45 degree C over the poles. The temperature here is nearly constant, and hence, it
is called the tropopause. The stratosphere is found above the tropopause and extends up to a height
of 50 km. One important feature of the stratosphere is that it contains the ozone layer. This layer
absorbs ultraviolet radiation and shields life on the earth from intense, harmful form of energy.

Statement 3 is not correct: The mesosphere lies above the stratosphere, which extends up to a
height of 80 km. In this layer, once again, temperature starts decreasing with the increase in
altitude and reaches up to minus 100°C at the height of 80 km. The upper limit of mesosphere is
known as the mesopause.

The thermosphere is the layer in the Earth's atmosphere directly above the mesosphere and below
the exosphere. It contains electrically charged particles known as ions, and hence, it is a part of
region known as the ionosphere (Ionosphere includes the thermosphere and parts of the
mesosphere and exosphere which are ionized by solar radiation). Radio waves transmitted from
the earth are reflected back to the earth by this layer. Temperature here starts increasing with
height.

The uppermost layer of the atmosphere above the thermosphere is known as the exosphere. This is
the highest layer but very little is known about it. Whatever contents are there, these are extremely
rarefied in this layer, and it gradually merges with the outer space.

Plot No: 935,6th Avenue, Anna Nagar, Chennai-40


Ph: 044-40483555,9677120226,9677174226
Web: www.officersiasacademy.com
OFFICERS IAS ACADEMY
(IAS Academy by IAS Officers)
Although all layers of the atmosphere must be exercising influence on us, geographers are
concerned with the first two layers of the atmosphere.

13. With reference to the pattern of wind circulation in the Indian Subcontinent, during summers,
which of the following statements are correct?
1. The wind circulation undergoes a complete reversal at the lower as well as the upperatmospheric
levels.
2. The westerly jet stream withdraws from the Indian region.
3. The Inter-Tropical Convergence Zone (ITCZ) shifts northwards.
Select the correct answer using the code given below.
(a) 1 and 2 only
(b) 2 and 3 only
(c) 1 and 3 only
(d) 1, 2 and 3
Ans D

Surface Pressure and Winds during the summer season in the Indian Subcontinent
As the summer sets in and the sun shift northwards, the wind circulation over the subcontinent
undergoes a complete reversal at both, the lower as well as the upper levels.
By the middle of July, the low-pressure belt nearer the surface [termed as Inter-Tropical
Convergence Zone (ITCZ)] shifts northwards, roughly parallel to the Himalayas between 20° N and
25° N.
By this time, the westerly jet stream withdraws from the Indian region.
The interrelationship between the northward shift of the equatorial trough (ITCZ) and the
withdrawal of the westerly jet stream from over the North Indian Plain.
It is generally believed that there is a cause and effect relationship between the two.
The ITCZ being a zone of low pressure attracts inflow of winds from different directions. The
maritime tropical airmass (mT) from the southern hemisphere, after crossing the equator, rushes
to the low pressure area in the general southwesterly direction. It is this moist air current which is
popularly known as the southwest monsoon.
Hence, all the statements are correct.

14. Consider the following pairs:


Ocean Current: Adjacent Coast
1. Agulhas Current: Eastern coast of Africa
2. Irminger Current: Coast of Alaska
3. Humboldt Current: Western coast of South America
Which of the pairs given above are correctly matched?
(a) 1 and 2 only
(b) 2 and 3 only
(c) 1 and 3 only

Plot No: 935,6th Avenue, Anna Nagar, Chennai-40


Ph: 044-40483555,9677120226,9677174226
Web: www.officersiasacademy.com
OFFICERS IAS ACADEMY
(IAS Academy by IAS Officers)
(d) 1, 2 and 3
Ans C

The Agulhas Current is the western boundary current of the South-West Indian Ocean. It flows
down the east coast of Africa. The source water at its northern end is derived from Mozambique
channel eddies and the East Madagascar Current, but the greatest source of water is recirculation in
the southwest Indian Ocean sub-gyre. Hence pair 1 is correctly matched.
The Irminger Current is a north Atlantic ocean current setting westward off the southwest coast of
Iceland. It is composed of relatively warm and saline waters from the eastern North Atlantic that
are fed by the North Atlantic Drift. The Irminger Current is part of the North Atlantic subpolar gyre.
Hence pair 2 is not correctly matched.
The Humboldt Current, also called the Peru Current, is a cold, low-salinity ocean current that flows
north along the western coast of South America. Hence pair 3 is correctly matched.

15. Consider the following statements


1. During the Neap Tides the high tide is lower and the low tide is higher than usual.
2. The Neap Tide, unlike the Spring Tide, occurs on the New Moon instead of on the Full Moon.
Select the correct answer
a) Only 1
b) Only 2
c) Both
d) Neither 1 nor 2

Answer: A

Tide – periodical rise and fall of the sea level, once or twice a day because of attraction of sun and
moon
 High tide – when water covers much of the shore by rising to its highest level.
 Low tide – When water falls to its lowest level and recedes from the shore.

Causes of tides
 To a great extent gravitational pull of moon
 To lesser extent gravitational pull of sun
 Centrifugal force - Force that acts to counterbalance the gravity
 Tide generating force is the difference between the gravitational attraction of the moon and
the centrifugal force of the earth.

Types of tides:
Spring tide – on full moon and new moon days, the sun, the moon and earth are almost in a same
line. On these days, the sun and the moon exert their combined gravitational force on the earth.
Thus on these two days the high tides are the highest and are known as spring tides. The height of a
spring tide is 20% more than the normal high tide.
Neap tide – on half-moon days (i.e. first and last quarter phases of the moon), the sun and the moon
are at 90° to the centre of the earth. The tide producing forces of the moon and the sun, work in
opposite directions and they partly cancel each other’s force. The height of the high tide is lower

Plot No: 935,6th Avenue, Anna Nagar, Chennai-40


Ph: 044-40483555,9677120226,9677174226
Web: www.officersiasacademy.com
OFFICERS IAS ACADEMY
(IAS Academy by IAS Officers)
than the normal high tide while low tide is higher than the normal low tide and the difference
is about 20%.

16. Which of the following statements clearly explains “breaks in the monsoon”?
(a) The date at which monsoon rainfall ends.
(b) It is the period in the South-West Monsoon during which rain fails to occur for days or weeks.
(c) It denotes the phenomenon when monsoon winds collide with the mountains of the Western Ghats
and the Himalayas.
(d) None of the above

Answer: (b)
During the south-west monsoon period, after having rains for a few days, if rain fails to occur for one or
more weeks, it is known as break in the monsoon. These dry spells are quite common during the rainy
season.
17. According to the Census 2011, which of the following states/UT has the highest male literacy rate in
India?
(a) Lakshadweep
(b) Mizoram
(c) Goa
(d) Kerala

Answer: (a)

Plot No: 935,6th Avenue, Anna Nagar, Chennai-40


Ph: 044-40483555,9677120226,9677174226
Web: www.officersiasacademy.com
OFFICERS IAS ACADEMY
(IAS Academy by IAS Officers)

18. Consider the following pairs:


1. National Waterway-1 - Haldia - Allahabad
2. National Waterway-2 - Kottapuram - Kollam
3. National Waterway-3 - Dhubri-Sadiya
Which of the pairs given above is/are correctly matched?
(a) 1 only
(b) 1 and 2 only
(c) 1 and 3 only
(d) 1, 2 and 3

Answer: (a)
Operational National Waterways in the country Out of the 111 National Waterways (NWs) declared
under the National Waterways Act, 2016, 13 NWs are operational for shipping and navigation, and
cargo/passenger vessels are moving on them.

Sl. Length
National Waterway (NW) No. Location (S)
No. (km)

NW-1: Ganga-Bhagirathi-Hooghly River System Uttar Pradesh, Bihar,


1. 1620
(Haldia - Allahabad) Jharkhand, West Bengal

2. NW-2: Brahmaputra River (Dhubri - Sadiya) 891 Assam

NW-3: West Coast Canal (Kottapuram - Kollam),


3. 205 Kerala
Champakara and Udyogmandal Canals

NW-4: Phase-1development of the stretch Muktiyala


4. 82 Andhra Pradesh
to Vijyawada of river Krishna

Plot No: 935,6th Avenue, Anna Nagar, Chennai-40


Ph: 044-40483555,9677120226,9677174226
Web: www.officersiasacademy.com
OFFICERS IAS ACADEMY
(IAS Academy by IAS Officers)

Waterways in Maharashtra
5. i) NW-10 (Amba River)
45
6. ii) NW-83 (Rajpuri Creek) Maharashtra
31
7. iii) NW-85 (Revadanda Creek - Kundalika River
31
System)
8. iv) NW-91 (Shastri river–Jaigad creek system)
52

National Waterways in Goa


9. NW-68 – Mandovi – Usgaon Bridge to Arabian Sea (41 41
km) Goa
10. NW-111 – Zuari– Sanvordem Bridge to Marmugao 50
Port (50 km).

National Waterways in Gujarat


11. NW-73- Narmada river- 226 Gujarat &Maharastra
12. NW-100- Tapi river 436

West Bengal
Sunderbans Waterways (NW-97): Namkhana to
13. 172 (through Indo-Bangladesh
AtharaBankiKhal in West Bengal.
Protocol Route)

19. Consider the following pairs regarding local winds:


1. Chinook – Canada
2. Foehn – Switzerland
3. Harmattan – Europe
Which of the pairs given above is/are correctly matched?
(a) 1 only
(b) 1 and 2 only
(c) 2 and 3 only
(d) 1, 2 and 3

Answer: (b)
Warm and dry winds blowing on the leeward sides of the mountains are called Chinook in the
Canada and Foehn in Switzerland (Alps). Warm and dry winds blowing from north-east and east to
west in the eastern parts of the Sahara Desert (Africa) are called Harmattan.

Plot No: 935,6th Avenue, Anna Nagar, Chennai-40


Ph: 044-40483555,9677120226,9677174226
Web: www.officersiasacademy.com
OFFICERS IAS ACADEMY
(IAS Academy by IAS Officers)
20. The Tropic of Cancer passes through which of the following states?
1. Uttar Pradesh
2. Rajasthan
3. Maharashtra
4. Tripura
Select the correct answer using the code given below:
(a) 1 and 3 only
(b) 2 and 4 only
(c) 1, 2 and 4 only
(d) 1, 2, 3 and 4

Answer: (b)
The Tropic of Cancer passes through the following 8 states:
Gujarat- Rajasthan- Madhya Pradesh- Chhattisgarh- Jharkhand- West Bengal- Tripura – Mizoram

Plot No: 935,6th Avenue, Anna Nagar, Chennai-40


Ph: 044-40483555,9677120226,9677174226
Web: www.officersiasacademy.com
OFFICERS IAS ACADEMY
(IAS Academy by IAS Officers)

HISTORY
1. Consider the following pairs:

Indus Valley Sites Location


1. Lothal Gujrat
2. Ropar Punjab
3. Mohenjodaro Rajasthan

Which of the pairs given above is/are correctly matched?

a) 1 and 2 only
b) 1 only
c) 2 and 3 only
d) 3 only

Answer (a)

Explanation
Lothal is located in Gujarat and Ropar is in Punjab. Mohenjodaro is located Pakistan.
Other important Cities:

Chanu-daro-Sindh, Pakistan
Kalibangan-Rajasthan
Banawali-Haryana
Rakhigarhi-Haryana

2. Consider the following statements

1. Dharmasutras and Smritis are considered part of religious literature.


2. Sangam literature does not constitute religious literature.
3. Rajatarangini was written by Bilhana about the dynasties of Kashmir.
4. The earliest inscriptions found, belong to the Harappan period.

Which among the above statements are correct?

Select the correct answer using the code given below.


(a) 1, 2 and 3 only
(b) 2 and 4 only
(c) 1 and 3 only
(d) 1, 2, 3 and 4

Plot No: 935,6th Avenue, Anna Nagar, Chennai-40


Ph: 044-40483555,9677120226,9677174226
Web: www.officersiasacademy.com
OFFICERS IAS ACADEMY
(IAS Academy by IAS Officers)
Answer: (b)

Explanation:
Dharmasutras, Smritis and Dharmashastras (commentaries) belong to a class of secular literature.
They are essentially lawbooks.

Sangam literature are secular in nature. They have much information about the social and economic
life of the Sangam Tamils. Pastoral wars, military exploits form the PURAM part whereas love,
separation and other aspects form the AKAM part.

Rajatarangini, written by Kalhana in 12th century, and it gives an account of the dynasties of
Kashmir.

Harappan seals are the earliest inscriptions found in India. They belong to the period of about 2500
BC. They have not been deciphered. They have a pictorial script.

3. Which of following does not belong to Orthodox school of Indian Hindu philosophy?

a) Samkhya
b) Ajivika
c) Vaisheshika
d) Mimaṃsa

Answer: (b)

Explanation:
Indian philosophy refers to ancient philosophical traditions of the Indian subcontinent. The
principal schools are classified as either orthodox or heterodox – āstika or nāstika – depending on
one of three alternate criteria: whether it believes the Vedas as a valid source of knowledge;
whether the school believes in the premises of Brahman and Atman; and whether the school
believes in afterlife and Devas.
There are six major schools of orthodox Indian Hindu philosophy—Nyaya, Vaisheshika,
Samkhya, Yoga, Mīmāṃsā and Vedanta, and five major heterodox schools—Jain, Buddhist, Ajivika,
Ajñana, and Cārvāka. However, there are other methods of classification; Vidyaranya for instance
identifies sixteen schools of Indian philosophy by including those that belong to the Śaiva and
Raseśvaratraditions.

4. Which one of the following books of ancient India has the love story of the son of the founder of
Sunga dynasty?

(a) Swapnavasavadatta
(b) Malavikagnimitra
(c) Meghadoota
(d) Ratnavali

Answer (b)

Plot No: 935,6th Avenue, Anna Nagar, Chennai-40


Ph: 044-40483555,9677120226,9677174226
Web: www.officersiasacademy.com
OFFICERS IAS ACADEMY
(IAS Academy by IAS Officers)
Explanation
It is a Sanskrit play by Kalidasa. It is based on some events of the reign of Pushyamitra
Shunga.
The play tells the story of the love of Agnimitra, the Shunga Emperor at Vidisha, for the beautiful
hand-maiden of his chief queen.

5. Which of the following statements is/are correct about Ibn Battuta?

1) He visited India during the 14th Century.


2) He visited and lived at the court of Firoz shah Tughlaq for 8 years.
3) He travelled all over the India and has left accounts even on plants, animals and condition of the
roads, etc.

a) 1 and 2 only
b) 2 and 3 only
c) 1 and 3 only
d) All the above

Answer: (c)

Explanation:
Ibn Battuta, a resident of Tangier in North Africa, visited India in the fourteenth century and lived at
the court of Muhammad bin Tughlaq for eight years. He travelled widely all over the India and has
left a very interesting account of products of the country, including fruits, flowers, herbs, etc., the
condition of the roads, and the life of the people.

6. Consider the following statements with regards to the period after the Revolt of 1857

1. The Zamindars were used as a counter-weight against the nationalist-minded intelligentsia.


2. The reactionary policy was extended in their attitude towards social reforms also.
3. The British gave active encouragement to social reactions.

Which of the above statements is/are correct?

(a) 1 only
(b) 1 and 2 only
(c) 2 and 3 only
(d) 1, 2 and 3

Answer: (d)

Explanation:
During the revolt, the peasant’s outburst of anger towards the landlords was tapped by the British
and they were used as allies. Their lands were restored. They were hailed as the traditional leaders
of the Indian people and their interests and privileges were protected at the cost of peasants.

Plot No: 935,6th Avenue, Anna Nagar, Chennai-40


Ph: 044-40483555,9677120226,9677174226
Web: www.officersiasacademy.com
OFFICERS IAS ACADEMY
(IAS Academy by IAS Officers)
Hence, they were used as counter-weight against the nationalist-minded intelligentsia. Hence
statement 1 is correct.

Since the British followed a policy of alliance with the conservative groups they abandoned the
previous policy of helping social reformers as they believed that it was a causal factor for the revolt.
Hence statement 2 is correct.

The British followed a divide and rule policy where it encouraged casteism and communalism for
political purposes. (Social reaction or social labelling is concerned with how the self-identity and
behaviour of individuals may be determined or influenced by the terms used to describe or classify
them). Therefore, they actively encouraged social reaction. Hence statement 3 is correct.

Source: Chapter 7 - Administrative changes after 1858, Modern India, NCERT, pg. 130.
7. Which of the following was not a reason for the formation of local bodies in 1860s?

(a) Industrial Revolution in Europe.


(b) Demand by nationalists to improve civic life.
(c) To channelize the political participation of Indians
(d) Social welfare motive of the Imperial Government

Answer: (d)

Explanation:
Financial difficulties after the great revolt led the colonial Indian government to further
decentralize administration by promoting local government through municipalities and district
boards. The industrial revolution gradually transformed the European society in 19th century.
India’s increasing contact with Europe and new modes of imperialism and economic exploitation
made it necessary that some of the European advances in economy, sanitation, education should be
transplanted in India. Hence option A is a factor for formation of local bodies.

The rising Indian nationalist movement because of the educated intelligentsia demanded the
introduction of modern improvements in civic life, need for education of the masses, sanitation,
water supply, better roads, and other civic amenities. Hence option B is also a reason for
introduction of local bodies.

The British believed that by associating Indians with the administration in some capacity or the
other would prevent their becoming politically disaffected. This association would not endanger the
British monopoly too. Hence, they wanted to involve Indians in administration from the local level.
Therefore, option C is a reason.

The British were not intrinsically motivated to improve the social conditions of Indians after 1857
revolt. Rather had other reasons like extra expenditure to not spend for their social welfare. but
they felt that the Indians who already are paying heavy taxes will be ready to pay more when they
know that it is for their own welfare. Hence option D was not an appropriate reason/intention
behind the introduction of local bodies.

Source: Chapter 7 - Administrative changes after 1858, Modern India, NCERT, pg. 124.

Plot No: 935,6th Avenue, Anna Nagar, Chennai-40


Ph: 044-40483555,9677120226,9677174226
Web: www.officersiasacademy.com
OFFICERS IAS ACADEMY
(IAS Academy by IAS Officers)

8. With reference to the Raja Jai Singh of Amber, consider the following statements

1. He erected observatories with accurate and advanced instruments.


2. He created a set of tables, titled ‘Zij Muhammadshahi’, to make astronomical observations.
3. He translated Euclid’s “Elements of Geometry” into Hindi.
4. He took steps to reduce female infanticide.

Which of the statements given above is/are correct?

a) 2 and 3 only
b)1, 2 and 3 only
c) 1, 3 and 4 only
d)1, 2 and 4 only

Answer: (d)

Explanation:
Jai Singh was a great astronomer.
He erected observatories with accurate and advanced instruments, some of them of his own
invention, at Delhi, Jaipur, Ujjain, Varanasi and Mathura.
His astronomical observations were remarkably accurate.
He created a set of tables, titled ‘Zij Muhammadshahi’, to enable people to make astronomical
observations.
Hence, the 1st and 2nd statements are correct.
He had Euclid’s “Elements of Geometry”, translated into Sanskrit as also several works on
trigonometry, and Napier’s work on the construction and use of algorithms.
Hence, the 3rd statement is incorrect.
Jai Singh was also a social reformer.
He tried to enforce a law to reduce the lavish expenditure which a Rajput had to incur on a
daughter’s wedding and which often led to infanticide.
Hence, the 4th statement is correct.

9. Which of the following reasons undermined the military strength of the Mughal Empire in the
18th century?

1. The nobles did not maintain their full quota of troops.


2. The civil wars between the nobles led to the death of many able commanders and soldiers.
3. The army lacked discipline and fighting morale.
4. The financial problems of the empire made it difficult to maintain a large army.

Select the correct answer using the code given below:

a) 1, 2 and 3 only

Plot No: 935,6th Avenue, Anna Nagar, Chennai-40


Ph: 044-40483555,9677120226,9677174226
Web: www.officersiasacademy.com
OFFICERS IAS ACADEMY
(IAS Academy by IAS Officers)
b) 1, 2 and 4 only
c) 1, 3 and 4 only
d) 1, 2, 3 and 4

Answer: (d)

All the statements are correct and are self-explanatory.

10. With respect to the incidents in the Civil Disobedience Movement, which of the following is in
the proper chronological order?

(a) Dandi March, 1st Round Table Conference, Gandhi Irwin Pact, Karachi session of Congress, 2nd
Round Table Conference
(b) Dandi March, Gandhi Irwin Pact, 1st Round Table Conference, Karachi session of Congress, 2nd
Round Table Conference
(c) Dandi March, 1st Round Table Conference, Gandhi Irwin Pact, 2nd Round Table Conference,
Karachi session of Congress
(d) Karachi session of Congress, Dandi March, Gandhi Irwin Pact, 1st Round Table Conference, 2nd
Round Table Conference

Answer: (a)

Explanation:
Dandi March - March 1930
First Round Table Conference –November 1930
Gandhi Irwin Pact - March 1931
Karachi session of Congress - endorsed Gandhi - Irwin Pact in March 1931
Second Round Table Conference - Aug-Sep 1931

11. Which of the following statements related to the Hinayana Buddhism sect is/are correct?

1. This sect believes in the heavenliness of Buddha and propagated Idol Worship.
2. Pali, the language of masses was used by the Hinayana scholars.
3. It is the dominant form of religion in Cambodia, Laos, Sri Lanka, Thailand, and Burma.

Which of the above statements is/are correct?

a) 1 and 2 only
b) 2 and 3 only
c) 3 only
d) All the above

Answer: (b)

Explanation:

Plot No: 935,6th Avenue, Anna Nagar, Chennai-40


Ph: 044-40483555,9677120226,9677174226
Web: www.officersiasacademy.com
OFFICERS IAS ACADEMY
(IAS Academy by IAS Officers)
It believes in the original teaching of Buddha or Doctrine of Elders. Does not believe in Idol
worship and tries to attain individual salvation through self-discipline and meditation.
Ultimate aim of Hinayana is thus nirvana.Statement 1 is incorrect.
Pali, the language of masses was used by the Hinayana scholars. Statement 2 is correct.
It is mainly practiced in Cambodia, Laos, Sri Lanka, Thailand, and Burma. Statement 3 is correct.
Source – Chapter no- 10 Ancient India NCERT XI

12. Consider the following pairs:

Newspaper Editor
Swadesamitran G. Subramaniya Iyer
Indian Mirror Dadabhai Naoroji
Advocate G P Varma

Which of the pairs given above is/are correctly matched?

(a) 1 only
(b) 1 and 3
(c) 3 only
(d) 2 and 3

Answer: (b)

Explanation:
The Hindu and Swadesamitran under the editorship of G. Subramaniya Iyer,
Indian Mirror under N.N. Sen,
Voice of India under Dadabhai Naoroji,
Hindustani and Advocate under G.P. Varma

13. Consider the following pairs:

Uprising Place

1. Namdhari Movement Maharashtra


2. Pabna Uprising Punjab
3. Ramosi Uprising Bengal

Which of the pairs given above is/are correctly matched?

(a) 1 only
(b) 1 and 2
(c) 2 and 3
(d) None

Answer: (d)

Explanation:

Plot No: 935,6th Avenue, Anna Nagar, Chennai-40


Ph: 044-40483555,9677120226,9677174226
Web: www.officersiasacademy.com
OFFICERS IAS ACADEMY
(IAS Academy by IAS Officers)
1. Namdhari Movement Punjab
2. Pabna Uprising Bengal
3. Ramosi Uprising Maharashtra

Vasudev Balwant Phadke, an educated clerk, raised a Ramosi peasant force in Maharashtra during
1879, and organized social banditry on a significant scale.

The Namdhari Movement was popularly known as Kuka Movement. It was founded in 1840 by
Bhagat Jawahar Mal (also called Sian Saheb) in western Punjab. A major leader of the movement
after him was Guru Ram Singh. (He founded the Namdhari Sikh sect.)

Pabna Peasant Uprising (1873-76) was a resistance movement by the peasants against the
zamindars in the Yusufshahi pargana in Pabna. It was led by Ishan Chandra Roy.

14. Which of the following are the economic impacts of British rule in India?

1. Stagnation and deterioration of agriculture


2. Commercialization of Indian agriculture
3. Rise of Indian Middle Class

Select the correct Answer using the code given below

(a) 1 only
(b) 2 and 3 only
(c) 1 and 2 only
(d) 1, 2 and 3

Answer: (d)

Explanation:
Burden on Land increased because of the increased number of people entering Agriculture. This led
to deterioration of agriculture. They also introduced oppressive middlemen to collect land revenue
from the peasants.
Statement 1 is Correct.
While the burden on agricultural land had increased, the British encouraged/forced farmers to
grow cash crops such as indigo instead of food grains. This changed the nature of crops sown and
the type of agriculture practiced in India.
Statement 2 is Correct.
With the increased opportunities in government services and growth of a few industries, the
middle-class section of the Indian society grew in number.

15. When Bakshi Jagabandhu Bidyadhar’s ancestral estate was taken by the British, what did it lead
to?

(a) Pagal Panthi Uprising

Plot No: 935,6th Avenue, Anna Nagar, Chennai-40


Ph: 044-40483555,9677120226,9677174226
Web: www.officersiasacademy.com
OFFICERS IAS ACADEMY
(IAS Academy by IAS Officers)
(b) Paika Rebellion
(c) Santhal rebellion
(d) Passing of Santhal Pargana Tenancy Act

Answer (b)

Explanation
The Paiks of Odisha were the traditional landed militia (‘foot soldiers’ literally) and enjoyed rent
free land tenures for their military service and policing functions on a hereditary basis. The EIC
dethroned their ruler
Bakshi Jagabandhu Bidyadhar had been the military chief of the forces. His ancestral estate of Killa
Rorang was taken over by the Company, reducing him to penury.
With active support of Mukunda Deva, the last Raja of Khurda, and other zamindars of the region,
Bakshi Jagabandhu Bidyadhar led a sundry army of Paikas forcing the East India Company forces to
retreat
Though the rebellion was brutally repressed by 1818, it succeeded in getting large remissions of
arrears, reductions in assessments, suspension of the sale of the estates of defaulters at discretion, a
new settlement on fixed tenures and other adjuncts of a liberal governance

Other revolts:
The PagalPanthi, are a semi-religious group mainly constituting the Hajong and Garo tribes (in
Bengal). It was founded by Karam Shah. Pagal Panthis refused to pay rent above a certain limit and
attacked the houses of zamindars. It was suppressed violently.
Santhals are a tribal group concentrated in the state of Jharkhand. The Santhal Revolt took place in
1855-56. The revolt can be attributed to the introduction of the Permanent Land Settlement of
1793.
The Santhal Pargana Tenancy Act 1949 was enacted after independence for the safety and special
identity of their region.

16. Which of the following was/were not the reason(s) for the rise of militant nationalism?

1. Partition of Bengal
2. Unemployment
3. Growth of self-reliance sentiments
4. Spread of Education

Select the answer using the code given below

(a) 1,2 and 3


(b) 2,3 and 4
(c) 1,3 and 4
(d) None of the above

Answer: (d)

Explanation:

Plot No: 935,6th Avenue, Anna Nagar, Chennai-40


Ph: 044-40483555,9677120226,9677174226
Web: www.officersiasacademy.com
OFFICERS IAS ACADEMY
(IAS Academy by IAS Officers)
The Militant nationalism was developing since the later decades of 19th century and early
20th century. The growing economic plight along with severe unemployment radicalised the
youth of the country. In the later half of 19th century, the Indian participation in education
increased by way of setting up of colleges and universities.This increased awareness of the british
policies was an indirect factor in the spread of extremist nationalism. The more exposure to
western principles by spread of education was largely influencing democracy, nationalism and
radicalism. The nationalist spirit preached by Lokmanya Tilak and others through press (Kesari
and Mahratta) taught the spirit of self reliance and self sacrifice. He became a living example of self
sacrifice by his actions against the British. The partition of Bengal was an immediate cause which
made the militant nationalists actions more explicit.
Hence all the above were the reasons for the rise of militant nationalism.
Source: Modern India, NCERT Class XII, Bipin Chandra, Pg no. 192.

17. Which of the following statements is/are true about the concept of separate electorate in British
India?

a) Separate electorate means some seats are reserved in legislature exclusively for depressed
classes.
b) Separate electorates for Muslims was introduced after the passage of Indian Council’s Act of
1909.
c) McDonald Award was meant to provide separate electorates to Anglo-Indians only.
d) None of the above

Answer: (b)

Explanation:
Separate electorate means some seats are reserved in legislature exclusively for depressed classes
for which only adults of these communities are eligible.
Separate electorates for Muslims was introduced after the passage of Morley- Minto reforms of
1909.
McDonald Award was meant to provide separate electorates of depressed classes.

18. The Salient Features of the Mont-Ford reforms are:

1) Dyarchy at the centre


2) Bicameral arrangement at the centre
3) Women were given the right to vote

Select the correct answer using the code given below

a) 1 and 2 only
b) 2 and 3 only
c) 1 and 3 only
d) 1, 2 and 3

Answer (b)

Plot No: 935,6th Avenue, Anna Nagar, Chennai-40


Ph: 044-40483555,9677120226,9677174226
Web: www.officersiasacademy.com
OFFICERS IAS ACADEMY
(IAS Academy by IAS Officers)
Explanation
The Main features of the Montague Chelmsford Reforms are-

Central Government
I)The Governor General was to be the chief executive authority.
II)The Governor General retained full control over the reserved subjects in the provinces .
III)A bicameral arrangement was introduced .

Provincial Government
I) Dyarchy i.e rule of two –executive councilors and popular ministers was introduced .
II) Governor was to be the executive head of the Province.
III) Subjects were divided into two lists – “reserved” which included subjects such as law and order
,finance, land revenue, irrigation etc. and “Transferred” subjects such as education ,health local
government ,industry ,agriculture ,excise etc.
IV)Ministers were to be responsible to the Legislature.
V) The Secretary of state and the Governor General could interfere in respect of “reserved” subjects
while in respect of the “Transferred” subjects , the scope for their interference was limited.
VI)Women were given the right to vote.
VII) The Legislative Councils could initiate legislation but the Governor’s assent was required.
VIII)The Governor could veto bills and issue ordinance

19. Which among the following statements is correct?

a) Congress participated only in one of the three Round Table Conferences.


b) The Civil Disobedience Movement was withdrawn completely in 1931
c) GOI Act 1935 provided for a Federation of provinces for the British provinces and it does not
include the Princely states.
d) The representatives from the Princely states ought to be partly elected and partly nominated.

Answer: (a)

Explanation:
Congress leaders were imprisoned during the Civil Disobedience Movement and the first RTC
(Round Table Conference) happened without them. They were released as part of Gandhi-Irwin
pact. They participated in the second RTC but the British govt refused to accept the Dominion status
demand. The third RTC (Nov 1932) led to the passing of the Government of India Act of 1935.
The Civil Disobedience Movement was temporarily suspended as part of Gandhi-Irwin pact in 1931,
to facilitate the release of the leaders and their participation in the second Round Table Conference.
It was later suspended in May 1933 and was withdrawn completely in May 1934.
The Government of India Act of 1935 provided for the establishment of an All India Federation,
based on the union of British Indian provinces as well as the Princely States.
The representatives of the Princely States were to be appointed/nominated directly by the Princes
and there was not provision of election to such posts.

Reference Page: 307, 309; History of Modern India, Bipan Chandra


Reference Page: 317; India's Struggle for Independence, Bipan Chandra and others
Reference Page: 309; History of Modern India, Bipan Chandra

Plot No: 935,6th Avenue, Anna Nagar, Chennai-40


Ph: 044-40483555,9677120226,9677174226
Web: www.officersiasacademy.com
OFFICERS IAS ACADEMY
(IAS Academy by IAS Officers)

20. Which of the following are related with Education during British Rule?

1. Charter Act (1813)


2. Wood’s Despatch (1854)

Select the correct Answer using the code given below

(a) 1 only
(b) 2 only
(c) Both 1 and 2
(d) Neither 1 nor 2

Answer: c) Both 1 & 2


Explanation
Charter Act 1813 - A sum of one lakh rupees was to be set aside for the revival, promotion and
encouragement of literature, learning and science among the natives of India, every year.
Statement 1 is Correct.
Wood’s despatch - Charles Wood, the President of the Board of Control of the English East India
Company, had a great impact on disseminating education in India when in 1854 he sent a despatch
to Lord Dalhousie, the then Governor-General of India and suggested that Primary Schools must
Adopt vernacular languages. He also suggested that high schools use anglo-vernacular medium and
that English should be the medium for college level education
Statement 2 is Correct.

Plot No: 935,6th Avenue, Anna Nagar, Chennai-40


Ph: 044-40483555,9677120226,9677174226
Web: www.officersiasacademy.com
OFFICERS IAS ACADEMY
(IAS Academy by IAS Officers)

ENVIRONMENT AND ECOLOGY


The topics given below can be referred in the respective months ‘Pulse’ magazine of Officer IAS
academy which can be downloaded from our website

https://officerspulse.com/monthly-pulse/

1. June 2019 (Topic name – Key themes under the topic)

a. Bangkok Declaration – ASEAN initiative - promises to prevent and significantly reduce


marine debris.

b. Bharat Stage norms – Bharat Stage (BS)-6 emission norms from 2020 – Pollution
reduction

c. Ranganathittu bird sanctuary – Karnataka - declared as “Important bird site” by Birdlife


International (a nonprofit organisation working for the avian conservation).

d. United Nations Convention to Combat Desertification (UNCCD) - India is hosted the


14th CoP of UNCCD in September 2019 in Delhi - Bonn Challenge - Forest landscape
restoration approach

e. Forest Rights Act (FRA) - In February, the Supreme Court ordered the eviction of lakhs
of people belonging to the Scheduled Tribes (STs) and Other Traditional Forest Dwellers
(OTFDs) categories across 16 States, whose claim as forest-dwellers has been rejected
under the Forest Rights Act.

f. Green Rating Project (GRP) - effort to rate industrial units within a specific sector on the
basis of their environment friendliness - Centre for Science and Environment’s initiative

g. Convention on Illegal Trade in Endangered Species (CITES) - India has sent a proposal to
CITES ahead of the upcoming CoP 18 that trade of individual species should be
regulated, and not the entire genus, based on their conservation status.

2. July 2019

a. Environmental and Social Management Framework – for Integrated coastal


management - unveiled by The Environment Ministry

Plot No: 935,6th Avenue, Anna Nagar, Chennai-40


Ph: 044-40483555,9677120226,9677174226
Web: www.officersiasacademy.com
OFFICERS IAS ACADEMY
(IAS Academy by IAS Officers)
b. Blue Flag Certification – The Union Environment Ministry has selected 12 beaches
in India to compete for the ‘Blue Flag’ certification.

c. Coastal Regulation Zone – The Supreme Court rejected an appeal against the
demolition of flats in Kerala built in violation of Coastal Zone Regulations 2018.

d. Plan Bee – an amplifying system imitating the buzz of a swarm of honey bees to keep
wild elephants away from railway tracks - strategy adopted by Northeast Frontier
Railway - earned the NFR the best innovation award in Indian Railways for the 2018-19
fiscal year.

e. Plastic Crust – layer of Polyethene/Polyethylene encrusted onto ocean rocks - a new


type of plastic pollution

f. Amrabad Tiger Reserve - A signature campaign for saving the Amrabad Tiger Reserve
from uranium mining has been launched

3. August 2019

a. SO2 emissions – A new report by the environmental NGO Greenpeace India shows that
India is the largest emitter of sulphur dioxide in the world.

b. National Clean Air Programme – launched by the Ministry of Environment, Forest and
Climate Change in January 2019 - aims to reduce particulate matter (PM2.5 and PM10)
pollution by 20-30% in 102 cities by 2024.

c. Deep ocean mission - Ministry Of Earth Sciences plans ‘Deep Ocean Mission’ to boost
India’s sea exploration capabilities.

d. Great Indian Bustard - nearing extinction due to collision with high voltage power lines
that criss-cross their flying path, according to a report by the Wildlife Institute of India
(WII), a statutory body under the MoEFCC.

4. September 2019

a. Forest-PLUS 2.0 – five-year programme that focuses on developing tools and


techniques to bolster ecosystem management and harnessing ecosystem services in
forest landscape management - US Agency for International Development (USAID) and
India's Ministry of Environment, Forest and Climate Change (MoEF&CC) initiative

Plot No: 935,6th Avenue, Anna Nagar, Chennai-40


Ph: 044-40483555,9677120226,9677174226
Web: www.officersiasacademy.com
OFFICERS IAS ACADEMY
(IAS Academy by IAS Officers)
b. The UN Climate Action Summit 2019 – held in September in New York.

c. Single use plastic – Prime Minister Narendra Modi said that the country should strive to
become free from single use plastics.

d. Cryosphere - The Intergovernmental Panel on Climate Change (IPCC 51) has released
the Special Report on the Ocean and Cryosphere in a Changing Climate (SROCC)

e. Paraquat Dichloride - Civil society groups in Odisha demanded the government to


include Paraquat Dichloride in the PIC list of the Rotterdam Convention.

f. India’s Cooling Action Plan - The United Nations applauded India’s Cooling Action Plan
as an important policy initiative which has the potential to provide socio-economic and
environmental benefits related to reduced refrigerant use, climate change mitigation
and Sustainable Development Goals (SDGs).

5. October 2019

a. RUCO - an initiative of FSSAI to enable collection and conversion of used cooking oil to
biodiesel

b. Green Wall of India - The centre is mulling an ambitious plan of creating Green Wall of
India.

6. November 2019

a. United Nations Emission Gap Report 2019 – The United Nations Environment
Programme has released its United Nations Emission Gap Report 2019
b. Avian botulism – The Indian Veterinary Research Institute, has confirmed avian
botulism as the reason for mass mortality of birds at Sambhar Lake in November, 2019

c. Swaach -Nirmal Tat Abhiyaan - launched by the Ministry of Environment forest and
Climate change (MoEFCC) to make beaches clean and create awareness among citizens
about the importance of coastal ecosystems.

d. Paris Climate Agreement - The United States has notified the United Nations of its
formal withdrawal from the Paris Agreement alleging that the agreement brings unfair
economic burden on Americans.

Plot No: 935,6th Avenue, Anna Nagar, Chennai-40


Ph: 044-40483555,9677120226,9677174226
Web: www.officersiasacademy.com
OFFICERS IAS ACADEMY
(IAS Academy by IAS Officers)
7. December 2019

a. Torrefaction - a thermal process to convert biomass into a coal-like material, which has
better fuel characteristics than the original biomass. - India is partnering with a Swedish
company to employ torrefaction in order to find a solution for stubble burning.

b. Pretilachlor - Pretilachlor is a commonly used herbicide - Scientists from Assam have


formed Carbon dots using water hyacinth to identify the presence of Pretilachlor in
water bodies.

8. January 2020

a. Senna spectabilis – It is an extremely fast-growing invasive species from tropical


America - Kerala forest and wildlife department is planning to adopt steps to arrest the
rampant growth of invasive plants, especially Senna spectabilis, in the forest areas of
the Nilgiri Biosphere Reserve.

b. HCFC-141 b - Hydrochlorofluorocarbon (HCFC)-141 b is a chemical used by foam


manufacturing enterprises and one of the most potent ozone depleting chemicals after
Chlorofluorocarbons (CFCs). - India has successfully completed the phasing out of
Hydrochlorofluorocarbon-141 b

c. Green credit Scheme - It is Scheme for reforestation - It allows the Forest Department
to outsource one of its responsibilities of reforesting to non-government agencies - The
Forest Advisory Committee has recommended the ‘Green Credit Scheme’.

9. February 2020

a. Convention on Migratory Species (CMS), also known as the Bonn Convention – India
hosted the 13th Conference of the Parties (COP 13) to the Convention on Migratory
Species in Gandhinagar, Gujarat.

b. Biorock or mineral accretion technology – The Zoological Survey of India (ZSI), with help
from Gujarat’s forest department, is attempting to restore coral reefs using biorock or
mineral accretion technology near the Gulf of Kachchh.

Plot No: 935,6th Avenue, Anna Nagar, Chennai-40


Ph: 044-40483555,9677120226,9677174226
Web: www.officersiasacademy.com
OFFICERS IAS ACADEMY
(IAS Academy by IAS Officers)
10. March 2020

a. Sukhna lake - The Punjab and Haryana High Court declared the Sukhna lake as a legal
person for preservation with corresponding rights, duties and liabilities of a living person
by invoking the parens patriae doctrine.

11. April 2020

a. Dehing Patkai Reserve - The Reserve consists of wet tropical evergreen Assam valley
forests. It is often referred to as "The Amazon of the east" owing to its large area and
thick forests - The National Board for Wildlife has given permission for coal mining in
Dehing Patkai Elephant Reserve.

b. Petersberg Climate Dialogue - The 11th edition of the Petersberg Climate Dialogue (PCD
XI) was held recently

12. June 2020

a. Permafrost – The principal reason that led to the recent 20,000-tonne oil leak at an
Arctic region power plant in Russia that is now being recognised is the sinking of ground
surface due to permafrost thaw.

Plot No: 935,6th Avenue, Anna Nagar, Chennai-40


Ph: 044-40483555,9677120226,9677174226
Web: www.officersiasacademy.com
OFFICERS IAS ACADEMY
(IAS Academy by IAS Officers)

ART AND CULTURE


The topics given below can be referred in the respective months ‘Pulse’ magazine of Officer IAS
academy which can be downloaded from our website

https://officerspulse.com/monthly-pulse/

1. June 2019 (Topic name – Key themes under the topic)

a. Chaukhandi Stupa – Buddhism and Jainism, Hiuen Tsang, ASI, protected area of
national importance.

b. Ambubachi Mela – Fairs and festivals, Temple Architecture of the East.

c. Amaravati school of Art – Various Indian Schools of Art (Sculpture), Satavahanas,


Ikshvakus, Jataka tales, Buddhism .

2. July 2019

a. The government is considering a proposal for granting Marathi the status of a


classical language – Classical Languages of India.

b. Palkhi Procession – Fairs and festivals, Dnyaneshwar and Tukaram (Bhakti


Saints/Bhakti Movement).

c. Jaipur declared as a UNESCO World Heritage Site – UNESCO World Heritage


Sites in India, UNESCO Tangible and Intangible Cultural Heritage List.

d. The UNESCO World Heritage Committee (WHC) has raised concerns about
developmental projects in the world heritage site of Hampi – Vijayanagara
Empire Art and Culture.

e. Archaeological Survey of India is carrying out restoration works of


Markandeshwar temple – Temple Architecture of India.

3. August 2019

a. GI Tags – Palani Panchamirtham

b. Iron Age site at Phupgaon – Iron Age and Vedic Age Art and Culture.

Plot No: 935,6th Avenue, Anna Nagar, Chennai-40


Ph: 044-40483555,9677120226,9677174226
Web: www.officersiasacademy.com
OFFICERS IAS ACADEMY
(IAS Academy by IAS Officers)

4. September 2019

a. Keezhadi – Sangam Age/Culture

b. Shondol dance – Folk Dances of India

c. Nuakhai Juhar – Fairs and Festivals

5. October 2019

a. Chavang Kut - Fairs and Festivals

b. Khon Ramlila - Fairs and Festivals, UNESCO’s Intangible Cultural Heritage.

c. Creative Cities Network - UNESCO Creative Cities Network (UCCN)

6. November 2019

a. Ganjifa – Ancient Indian Games

b. Parliament Building – Modern Indian Architecture

7. December 2019

a. Losar festival - Fairs and Festivals

8. January 2020

a. The Belum Caves – ASI Monuments of National Importance

9. February 2020

a. Yakshagana – Indian Theatre, Bhakti movement

b. Konark Sun Temple – Ancient Indian Architecture, Kalinga architecture,

c. Brihadeeswarar Temple - UNESCO World Heritage Site, Temple Architecture

d. LUI-NGAI-NI - Fairs and Festivals

Plot No: 935,6th Avenue, Anna Nagar, Chennai-40


Ph: 044-40483555,9677120226,9677174226
Web: www.officersiasacademy.com
OFFICERS IAS ACADEMY
(IAS Academy by IAS Officers)
e. Dara Shukoh – Mughal Rule, Indian Literature

10. March 2020

a. Chapchar Kut - Fairs and Festivals

11. April 2020

a. Meru Jatra festival - Fairs and Festivals

12. May 2020

a. Purandara Dasa – Indian Music, Bhakti movement

b. GI Tags - Thanjavur Netti Works, Arumbavur wood carvings from Tamilnadu,


Jharkhand’s Sohrai Khovar painting and Telangana’s Telia Rumal

13. June 2020

a. Pashupatinath Temple – Indian Temple Architecture, UNESCO World Heritage site

b. Talamaddale – Indian Theatre

c. Jagannath Temple, Puri - Indian Temple Architecture, Kalinga or Odisha school

d. Kodumanal megalithic site – Megaliths, Iron Age, Sangam Age

e. Saindhava empire – Late Ancient/Early Medieval Empires

14. July 2020

a. Zardozi embroidery – Indian Handicrafts

b. Sree Padmanabhaswamy Temple – Indian Temple Architecture, Bhakti Movement

15. August 2020

a. Pandit Jasraj – Indian Music, Hindustani, Khayal, Ghazal, Gharana


b. Behrupiya – Folk Arts
c. Pulikali – Folk Dances

Plot No: 935,6th Avenue, Anna Nagar, Chennai-40


Ph: 044-40483555,9677120226,9677174226
Web: www.officersiasacademy.com

You might also like